Инфоурок Математика Другие методич. материалыМЕТОДИЧЕСКИЕ УКАЗАНИЯ по выполнению практических работ по дисциплине ОУД.03 «Математика: алгебра и начала математического анализа, геометрия» 1 курс специальность 35.02.01

МЕТОДИЧЕСКИЕ УКАЗАНИЯ по выполнению практических работ по дисциплине ОУД.03 «Математика: алгебра и начала математического анализа, геометрия» для студентов 1 курса специальности 35.02.01 «Лесное и лесопарковое хозяйство»

Скачать материал


ДЕПАРТАМЕНТ ЛЕСНОГО ХОЗЯЙСТВА НИЖЕГОРОДСКОЙ ОБЛАСТИ

Государственное бюджетное профессиональное

образовательное учреждение Нижегородской области

«КРАСНОБАКОВСКИЙ ЛЕСНОЙ КОЛЛЕДЖ»






УТВЕРЖДАЮ

Зам. директора по учебной работе

____________________О.Н.Спирин







МЕТОДИЧЕСКИЕ УКАЗАНИЯ

по выполнению практических работ

по дисциплине ОУД.03 «Математика: алгебра и начала математического

анализа, геометрия»

для студентов 1 курса специальности

35.02.01 «Лесное и лесопарковое хозяйство»




РАССМОТРЕНО

на заседании предметно-цикловой комиссии

общеобразовательных дисциплин

протокол №___от « »___________2017г.

Председатель

________________И.А. Шарова


СОСТАВИЛА

Преподаватель

Г.А. Чудоквасова











р.п. Красные Баки

2017г



СОДЕРЖАНИЕ

1. Требования к знаниям и умениям при выполнении практических работ

3

2. Правила выполнения практических работ

5

Практическая работа №1.. Приближенные вычисления.

6

Практическая работа № 2. Арифметические операции над комплексными числами.

13

Практическая работа № 3. Основное логарифмическое тождество. Правила действий с логарифмами.

18

Практическая работа № 4. Преобразование алгебраических выражений различных типов.

20

Практическая работа № 5. Решение тригонометрических уравнений.

26

Практическая работа № 6. Решение тригонометрических неравенств.

32

Практическая работа № 7. Решение тригонометрических уравнений и неравенств.

34

Практическая работа №8. Построение графиков функций, заданных различными способами.

35

Практическая работа № 9. Преобразование графиков.

38

Практическая работа №10. Вычисление пределов.

43

Практическая работа № 11. Нахождение производной функции.

48

Практическая работа № 12. Использование производной для нахождения наилучшего решения в прикладных задачах.

51

Практическая работа № 13. Практическая работа №13.

56

Практическая работа № 14. Применение производной к исследованию функций и построению графиков.

60

Практическая работа № 15-16. Нахождение неопределенного интеграла.

64

Практическая работа №17. Нахождение площади криволинейной трапеции.

66

Практическая работа № 18. Вычисление интегралов. Нахождение площади криволинейной трапеции.

69

Практическая работа №19. Решение уравнений и неравенств.

Практическая работа № 20-21. Решение задач на перебор вариантов, на подсчет числа размещений, перестановок, сочетаний.

73

76

Практическая работа № 22. Дискретная случайная величина, закон ее распределения.

79

Практическая работа № 23. Решение практических задач с применением вероятностных методов.

85

Практическая работа № 24. Изображение пространственных фигур.

89

Практическая работа № 25. Представление о правильных многогранниках. Сечение куба, призмы, пирамиды.

98

Практическая работа № 26. Представление о правильных многогранниках. Сечение куба, призмы, пирамиды.

104

Практическая работа № 27. Решение задач на нахождение объемов тел и площадей их поверхностей.

105

Практическая работа № 28. Векторы.

108

Практическая работа № 29 -30. Использование координат и векторов при решении математических и прикладных задач.

111

Литература

115









Введение

В соответствии с ФГОС дисциплина ОУД.03 «Математика: алгебра и начала математического анализа, геометрия» по специальности 35.02.01 «Лесное и лесопарковое хозяйство» входит в общеобразовательный цикл, формирует базовые знания для освоения общепрофессиональных и специальных дисциплин.

Целью данных методических рекомендаций является организация преподавателем эффективной работы студентов на практических занятиях по дисциплине ОУД.03 «Математика: алгебра и начала математического анализа, геометрия» как средства, способствующего повышению качества образовательного процесса.

Задачи:

  1. сформировать общие и профессиональные компетенции во время проведения практических работ через содержание представленных методических рекомендаций;

  2. помочь преподавателю в подборе материала предлагаемого студентам для выполнения практических работ с целью закрепления и углубления знаний;

  3. рационально организовать самостоятельную работу студентов по выполнению практических работ через распределение времени, затраченного на ее выполнение, предложенную форму контроля их знаний, критерии оценок.

Практическая работа является одним из видов учебных занятий студентов, выполняемых под руководством преподавателя.

Основные цели практических работ:

- систематизация и закрепление знаний и практических умений студентов полученных при изучении на уроке;

- углубление и расширение теоретических знаний, формирование умений использовать справочную документацию, дополнительную литературу;

- развитие познавательных способностей и активности студентов, творческой инициативы, самостоятельности, ответственности и организованности;

- формирование самостоятельного мышления;

- развитие исследовательских умений.

1. Требования к знаниям и умениям при выполнении практических работ

В результате работ, предусмотренных программой по данной специальности, студент должен

знать:

  • значение математической науки для решения задач, возникающих в теории и практике; широту и в то же время ограниченность применения математических методов к анализу и исследованию процессов и явлений в природе и обществе;

  • значение практики и вопросов, возникающих в самой математике для формирования и развития математической науки; историю развития понятия числа, создания математического анализа, возникновения и развития геометрии;

  • универсальный характер законов логики математических рассуждений, их применимость во всех областях человеческой деятельности;

  • вероятностный характер различных процессов окружающего мира.

умет:

  • выполнять арифметические действия над числами, сочетая устные и письменные приемы; находить приближенные значения величин и погрешности вычислений (абсолютная и относительная); сравнивать числовые выражения;

  • находить значения корня, степени, логарифма, тригонометрических выражений на основе определения, используя при необходимости инструментальные средства; пользоваться приближенной оценкой при практических расчетах;

  • выполнять преобразования выражений, применяя формулы, связанные со свойствами степеней, логарифмов, тригонометрических функций;

  • вычислять значение функции по заданному значению аргумента при различных способах задания функции;

  • определять основные свойства числовых функций, иллюстрировать их на графиках;

  • строить графики изученных функций, иллюстрировать по графику свойства элементарных функций;

  • использовать понятие функции для описания и анализа зависимостей величин;

  • находить производные элементарных функций;

  • использовать производную для изучения свойств функций и построения графиков;

  • применять производную для проведения приближенных вычислений, решать задачи прикладного характера на нахождение наибольшего и наименьшего значения;

  • вычислять в простейших случаях площади и объемы с использованием определенного интеграла;

  • решать рациональные, показательные, логарифмические, тригонометрические уравнения, сводящиеся к линейным и квадратным, а также аналогичные неравенства и системы;

  • использовать графический метод решения уравнений и неравенств;

  • изображать на координатной плоскости решения уравнений, неравенств и систем с двумя неизвестными;

  • составлять и решать уравнения и неравенства, связывающие неизвестные величины в текстовых (в том числе прикладных) задачах;

  • решать простейшие комбинаторные задачи методом перебора, а также с использованием известных формул;

  • вычислять в простейших случаях вероятности событий на основе подсчета числа исходов;

  • распознавать на чертежах и моделях пространственные формы; соотносить трехмерные объекты с их описаниями, изображениями;

  • описывать взаимное расположение прямых и плоскостей в пространстве, аргументировать свои суждения об этом расположении;

  • анализировать в простейших случаях взаимное расположение объектов в пространстве;

  • изображать основные многогранники и круглые тела; выполнять чертежи по условиям задач;

  • строить простейшие сечения куба, призмы, пирамиды;

  • решать планиметрические и простейшие стереометрические задачи на нахождение геометрических величин (длин, углов, площадей, объемов);

  • использовать при решении стереометрических задач планиметрические факты и методы;

  • проводить доказательные рассуждения в ходе решения задач.

  • использовать приобретенные знания и умения в практической деятельности и повседневной жизни:

  • для практических расчетов по формулам, включая формулы, содержащие степени, радикалы, логарифмы и тригонометрические функции, используя при необходимости справочные материалы и простейшие вычислительные устройства;

  • для описания с помощью функций различных зависимостей, представления их графически, интерпретации графиков;

  • решения прикладных задач, в том числе социально-экономических и физических, на наибольшие и наименьшие значения, на нахождение скорости и ускорения;

  • для построения и исследования простейших математических моделей;

  • для анализа реальных числовых данных, представленных в виде диаграмм, графиков;

  • анализа информации статистического характера;

  • для исследования (моделирования) несложных практических ситуаций на основе изученных формул и свойств фигур;

  • вычисления объемов и площадей поверхностей пространственных тел при решении практических задач, используя при необходимости справочники и вычислительные устройства.

Изучение данной учебной дисциплины направлено на формирование следующих общих компетенций (ОК) обучающихся:

ОК 1. Понимать сущность и социальную значимость своей будущей профессии, проявлять к ней устойчивый интерес.

ОК 2. Организовывать собственную деятельность, выбирать типовые методы и способы выполнения профессиональных задач, оценивать их эффективность и качество.

ОК 3. Принимать решения в стандартных и нестандартных ситуациях и нести за них ответственность.

ОК 4. Осуществлять поиск и использование информации, необходимой для эффективного выполнения профессиональных задач, профессионального и личностного развития.

ОК 5. Использовать информационно-коммуникационные технологии в профессиональной деятельности.

ОК 6. Работать в коллективе и в команде, эффективно общаться с коллегами, руководством, потребителями.

ОК 7. Брать на себя ответственность за работу членов команды (подчиненных), результат выполнения заданий.

ОК 8. Самостоятельно определять задачи профессионального и личностного развития, заниматься самообразованием, осознанно планировать повышение квалификации.

ОК 9. Ориентироваться в условиях частой смены технологий в профессиональной деятельности.


2. Правила выполнения практических работ.

В начале учебного года (на первом учебном занятии) преподаватель знакомит студентов со структурой построения всего курса дисциплины ОУД.03 «Математика: алгебра и начала математического анализа, геометрия», в которую должны быть органично вписаны практические работы. Каждый студент после такого занятия должен понимать, сколько практических работ ему предстоит выполнить в период изучения дисциплины и, каким образом он будет отчитываться перед преподавателем. Можно составить таблицу, по которой студенту легко будет ориентироваться по темам курса, видам практических работ, срокам выполнения, критериям оценивания.

Правила выполнения практических работ

  1. Каждый студент после выполнения работы должен представить отчет о проделанной работе с анализом полученных результатов и выводом по работе.

  2. Отчет о проделанной работе следует делать в тетради школьного формата в клетку. Содержание отчета указано в описании практической работы.

  3. Таблицы и рисунки следует выполнять с помощью чертежных инструментов (линейки, циркуля и т. д.) карандашом с соблюдением ЕСКД.

  4. Расчет следует проводить с точностью до двух значащих цифр. Вспомогательные расчеты можно выполнить на отдельных листах, а при необходимости на листах отчета.

5. Если студент не выполнил практическую работу или часть работы, то он может выполнить работу или оставшуюся часть во внеурочное время, согласованное с преподавателем.

6. Критериями оценки результатов практической работы студентов являются:

  • уровень усвоения студентом учебного материала;

  • умение студента использовать теоретические знания при выполнении практических задач;

  • сформированность общеучебных умений;

  • обоснованность и четкость изложения материала;

  • уровень оформления работы.

Зачет по практическим работам студент получает при условии выполнения всех предусмотренной программой работ, после сдачи отчетов по работам при удовлетворительных оценках за опросы и контрольные вопросы во время практических занятий.

На выполнение практических работ в курсе изучения дисциплины отводится 102 часа. Методические рекомендации помогут студентам целенаправленно изучать материал по теме, определять свой уровень знаний и умений при выполнении практической работы.


3. Основная часть:

Практическая работа №1.

Тема: Приближенные вычисления

Цель: сформировать у студентов знания, умения и навыки работы с приближенными числами в применении формул погрешностей элементарных действий и функций, нахождения значений выражений по способу границ и методом строгого учета абсолютных погрешностей после каждой операции.

Форма организации занятия – индивидуальная.

Студент должен

знать:

  • определение приближенного числа;

  • определение погрешности.

уметь:

  • вычислять абсолютную и относительную погрешность;

  • применять формулы погрешности арифметический действий;

  • применять формулы погрешности элементарных функций.

Содержание отчета о работе:


  1. Отчет должен содержать:

  • указание темы практической работы;

  • цели;

  • формулировку задания.

  1. Вариант определяется согласно контрольному списку.

  2. Все задания (1-3) выполняются письменно, указываются результаты всех промежуточных вычислений.

  3. При оценивании учитывается правильность и аккуратность выполнения задания.


Методические указания:

ех – абсолютная погрешность.

δх – относительная погрешность.

х – точное значение величины.

- приближенное значение величины (приближение)


ех = |х - |


Пример 1. Дано число х=0,00006 и его приближение =0,00005. Найти абсолютную и относительную погрешности приближения.

Решение: ex = | 0,00006-0,00005| = 0,00001

Ответ: абсолютная погрешность 0,00001 и относительная погрешность приближения равна 20%


Пример 2. Найти предельные абсолютные и относительные погрешности числа х = 984,6, если оно имеет только верные цифры в строгом смысле.

Решение:

Цифры числа верны в строгом смысле, если абсолютная погрешность данного числа не превосходит половины единицы разряда, в котором записана последняя верная цифра числа.

( т.к. 6 –последняя верная цифра, стоит в разряде десятых)



Ответ: абсолютная погрешность для числа х ех=0,05

относительная погрешность числа х δх=0,0051

Пример 3. Найти предельные абсолютные и относительные погрешности числа х =2,364, если оно имеет только верные цифры в широком смысле.

Решение:

Цифры числа верны в широком смысле, если абсолютная погрешность данного числа не превосходит единицы разряда, в котором записана последняя верная цифра числа.

ех = 0,001 (последняя цифра 4 - разряд тысячных)


Ответ: абсолютная погрешность для числа х ех = 0,001

относительная погрешность числа х δх = 0,0423%.


Погрешность округленного числа.

Пример 4: Округляя число х=1,1426 до четырех значащих цифр, определить абсолютную и относительную погрешности полученных приближений. Цифры верны в широком смысле.

Решение:

По определению верной цифры в широком смысле абсолютная погрешность ех=0,0001

Округлим число х до четырех значащих цифр: х1=1,143

Погрешность округленного числа равна сумме погрешности исходного числа и погрешности округления:

Δокр=| 1,143-1,1426| = 0,0004

ех1= 0,0004+0,0001=0,0005



Пример5: Число х, все цифры которого верны в строгом смысле округлить до трех значащих цифр после запятой. Для полученного результата х1 вычислить границу абсолютной и относительной погрешностей. В записи числа х1 указать количество верных цифр погрешности. х=1,1426

Решение:


х1=1,143

ех1= ех + Δокр

Δокр= | 1,143-1,1426| = 0,0004

ех1= 0,00005+0,0004=0,00045<0,0005

Значит в числе 1,143 цифры верны в строгом смысле до тысячных по абсолютной погрешности.



Вычислительная погрешность

1. Погрешность суммирования чисел х ± ех, у±еу

Абсолютная погрешность:

z =( х ± ех)+ (у±еу)=(x + y) ± ( ех + еу)

Относительная погрешность:



2. Погрешность вычитания чисел х ± ех, у±еу

Абсолютная погрешность:

z =( х ± ех)- (у±еу)=(x - y) ± ( ех + еу)

Относительная погрешность:



3. Погрешность умножения чисел х ± ех, у±еу

Абсолютная погрешность:

z =( х ± ех)* (у±еу)=ху±уех±хеу±ехеу ≈ ху±уех±хеу

Относительная погрешность:



4. Погрешность деления чисел х ± ех, у±еу

Абсолютная погрешность:


Относительная погрешность:


Погрешности элементарных функций.

Погрешность функции, зависящей от одной переменной.

Абсолютная погрешность:

f(х ± ех) ≈ f(x) ± f (x)ex

Δf = f(х ± ех) - f(х)=| f (х)|ex

Относительная погрешность:

Пример 6: Исходные числовые значения аргумента заданы цифрами, верными в строгом смысле. Найти абсолютную и относительную погрешности функции f(x)=cos(0,47). Определить количество верных цифр в строгом смысле по относительной погрешности. В ответе сохранить верные цифры и одну сомнительную.

Решение:

Найдем значение функции f(x)=cos(0,47)=0,891568

абсолютная погрешность:

Δf = | f (х)|ex

1) | f (х)| = sin(0,47)=0,452886285

2) e0,47=0,005

3) Δf=0,452886285*0,005=0,00226443.


Относительная погрешность:



Значит в числе 0,891568 две цифры после запятой верны в строгом смысле.

Ответ: 0,892

Пример 7. Вычислить значение величины с помощью метода строгого учета границ абсолютных погрешностей после каждой операции, цифры верны в строгом смысле.

, если а = 12,34, b= 14,3

Решение:

Для получения значения величины А необходимо выполнить 6 действий. Будем вычислять абсолютную погрешность после каждого действия с целью определения количества верных цифр в промежуточных результатах.

При пооперационном строгом учете ошибок промежуточные результаты после округления до одной запасной (с учетом вычисленной параллельно величины погрешности) и их погрешности заносят в таблицу

Значения погрешностей для удобства округлим до двух значащих цифр по избытку и тоже занесем в таблицу.

Цифры даны верными в строгом смысле, значит еа=0,005, ев=0,05

Найдем

Абсолютная погрешность равна

Из полученного значения погрешности видно, что в результате верны две значащие цифры после запятой, т.е.

( сохраняем одну сомнительную цифру)


Найдем

Абсолютная погрешность равна


Из полученного значения погрешности видно, что в результате верна одна значащая цифра после запятой, т.е.

( сохраняем одну сомнительную цифру)


Найдем

z =( х ± ех)+ (у±еу)=(x + y) ± ( ех + еу)= (3,513+3,78) ± (0,00071+0,0066) = 7,293 ± 0,00731

т.к. 0,00731<=0,05, то в числе 7,293 одна верная цифра после запятой, т.е. 7,293 ≈ 7,29( сохраняем одну сомнительную цифру)


Найдем ln(a)= ln(12,34)=2,51285

Абсолютная погрешность:

В числе 2,512846 верны три значащие цифры после запятой, т.е.

ln(12,34)=2,512846 ≈ 2,5128( сохраняем одну сомнительную цифру)


Найдем b + ln(a ) = (14,3 + 2,5128 ) ± (0,05+0,00041) = 16,8128 ± 0,050405

Т.к. , то в числе 16,8128 верны цифры до единиц 16,8128 ≈ 16,8 (сохраняем одну сомнительную цифру)

Найдем А




Округлим результат А до двух верных цифр после запятой, получим окончательный ответ: А=0,434 (сохраняем одну сомнительную цифру)

Ответ: А = 0,434 ± 0,002


Погрешности значений элементарных функций.

Таблица 1





Задания для практического занятия №1.


Задание №1.

Найти предельные абсолютные и относительные погрешности чисел, если они имеют только верные цифры:

а) в строгом смысле; б) в широком смысле.

варианта

а)

б)


варианта

а)

б)

1

11,445

2,043

16

112,5

0,04453

2

8,345

0,288

17

0,576

2,5008

3

0,374

4,348

18

25,613

0,0748

4

41,72

0,678

19

0,4223

0,57

5

18,357

2,16

20

112,45

3,4

6

14,862

8,73

21

2,4516

0,863

7

0,3648

21,7

22

5,6432

0,00858

8

0,5746

236,58

23

12,688

4,636

9

5,634

0,0748

24

15,644

6,125

10

20,43

0,576

25

16,383

5,734

11

12,45

3,4453

26

18,275

0,00644

12

2,3445

0,745

27

3,75

6,8343

13

0,5746

42,884

28

26,3

4,8556

14

3,4

0,078

29

43,813

0,645

15

2,4342

0,57004

30

3,643

72,385


Задание №2.

Число х, все цифры которого верны в строгом смысле, округлить до трех значащих цифр. Для полученного результата х1≈х вычислить границы абсолютной и относительной погрешностей. В записи числа х1 указать количество верных цифр по погрешности.



Задание № 3

Вычислить значение величины Z при заданных значениях чисел a,b,c используя систематический учет абсолютных погрешностей после каждой операции, а также с помощью метода границ. Найти абсолютную и относительную погрешности z и определить по ним количество верных цифр в z, если цифры a,b,c верны в строгом смысле.

варианта

Задание

Исходные данные

варианта

Задание

Исходные данные

1


a = 0,0399

b = 4,83

c = 0,0721

16

z = a2+sin(b-ln(c))

a =8,317

b = 13,521

c = 6,123

2


a =5,52

b =3,27

c =14,123

17


a = 0,038

b = 3,9353

c = 5,75

3


a =2,258

b =0,027

c =9,87

18


a = 7,345

b = 0,31

c = 0,09871

4


a =1,0574

b =1,40

c =1,1236

19


a =0,2471

b =0,0948

c =4,378

5


a =3,49

b =0,845

c =0,0037

20


a = 1,284

b = 4,009

c = 3,2175

6


a =0,0976

b =2,371

c =1,15887

21


a = 18,407

b = 149,12

c = 2,3078

7


a =82,3574

b =34,12

c =7,00493

22


a = 29,49

b = 87,878

c = 4,403

8


a =3,71452

b =3,03

c =0,756

23


a = 74,079

b = 5,3091

c = 6,234

9


a =0,11587

b =4,256

c =3,00971

24


a =3,4

b =6,22

c =0,149

10


a = 4,05

b = 6,723

c = 0,03254

25


a =5,387

b =13,527

c =0,7565

11


a = 0,7219

b = 135,347

c =0,013

26


a = 1,75

b = 1,215

c = 0,041

12


a = 0,113

b = 0,1056

c = 89,4

27


a =3,672

b =4,63

c =0,0278

13


a = 1,247

b = 0,346

c = 0,051

28


a =0,317

b =13,57

c =0,751

14


a = 18,035

b = 3,7251

c = 0,071

29


a =0,317

b =33,827

c =14,85

15


a = 0,317

b = 3,27

c = 4,7561

30


a =12,72

b =0,34

c =0,0290

Вопросы по теме:

  1. Что такое абсолютная и относительная погрешности?

  2. Как классифицируют виды погрешностей?

  3. Что значит цифра, верная в строгом, широком смыслах?

  4. Как находится погрешность округленного числа?

  5. Как определить количество верных цифр по абсолютной погрешности.

Практическая работа № 2

Тема: Арифметические операции над комплексными числами.

Цель: сформировать умение выполнять арифметические действия с комплексными числами.

Методические указания для практической работы

Теоретические сведения к практической работе

1. Понятие комплексного числа

Комплексными числами называются числа вида

, (1.1)

где x, y – действительные (вещественные) числа, а число i определяемое равенством = – 1 (), называется мнимой единицей.

Число x называется действительной (вещественной) частью комплексного числа (используется обозначение ); y – мнимой частью комплексного числа z ().

Выражение (1.1) называют алгебраической формой записи комплексного

числа.

Если x=0, то число z называют чисто мнимым; если , то получается вещественное число .

Два комплексных числа и называются сопряженными. Используя формулу разности квадратов, получаем, что .

Можно доказать, что корнями квадратного уравнения с отрицательным дискриминантом являются два сопряженных комплексных числа.

Пример 1. Решить уравнение .

Решение. Дискриминант данного уравнения: меньше нуля, но теперь мы можем воспользоваться мнимой единицей:

, т.е. ; .

Два комплексных числа и равны друг другу, если и ; комплексное число z считается равным нулю, если .

Всякое комплексное число можно изобразить на плоскости, т.к. каждому z соответствует упорядоченная пара вещественных чисел:hello_html_27ccf61f.gif



х

Число z=0 ставится в соответствие началу координатной плоскости. Такую плоскость мы в дальнейшем будем называть комплексной плоскостью, ось абсцисс–действительной, а ось ординат–мнимой осью комплексной плоскости.

Число называется модулем комплексного числа и обозначается или .


2. Тригонометрическая форма комплексного числа Тригонометрическая форма комплексного числа. Каждому комплексному числу вида (1.1) можно поставить в соответствие точку M(x;y) на декартовой плоскости (при этом на оси Oх располагаются вещественные числа , а на оси OY – чисто мнимые числа ).

Модулем комплексного числа назовем длину отрезка (или расстояние от начала координат до точки M), т.е. . Аргументом комплексного числа () назовем угол, который вектор образует с положительным направлением оси Oх. Главное значение аргумента, которое, как правило, используется при осуществлении действий с комплексными числами, удовлетворяет условию .

При этом выражение вида

(1.2)

называется тригонометрической формой записи комплексного числа.

Преобразуем (1.1)


и, сравнивая с (1.2), получаем, что φ – аргумент комплексного числа z можно найти, решив систему

или (1.3)

Заметим, что при выборе значений φ из последнего уравнения необходимо учитывать знаки x и y.

φ – аргумент комплексного числа z можно найти формул , (1.3) или в силу того, что , .


Пример 2. Записать комплексное число в тригонометрической форме:

  1. 6i; b) , указать модуль и аргумент комплексного числа.

Решение. a) Здесь х=0, у=6 .

Поскольку число 6i лежит на положительной полуоси Оу, то значение аргумента , поэтому .

  1. Здесь х=1, у= .

По определению . Для определения аргумента воспользуемся формулой: . Получаем, что . Тригонометрическая форма заданного комплексного числа имеет вид: .

Пример 3. Записать в тригонометрической форме комплексное число .

Решение. Найдем модуль и аргумент комплексного числа: . Угол φ найдем из соотношений , . Тогда получим . Очевидно, точка находится во второй четверти: .

Подставляя в формулу (1.2) найденные r и φ, имеем .


3. Действия над комплексными числами

1) Сумма двух комплексных чисел и определяется согласно формуле .

2) Операция вычитания комплексных чисел определяется как операция, обратная сложению. Комплексное число , если , является разностью комплексных чисел z1 и z2. Тогда .

Пример 4. Выполнить действия: а) (4+2i)+(1+5i); b) (3+5i)-(6+3i).

а) По правилу сложения комплексных чисел получим

(4+2i)+(1+5i)=(4+1)+(2+5) i =5+7i.

b) По правилу вычитания комплексных чисел получим

(3+5i)-(6+3i)=(3-6)+(5-3)i=-3+2i.

3) Произведение двух комплексных чисел и определяется по формуле . В частности.

Можно получить формулу умножения комплексных чисел в тригонометрической форме.

Имеем .

Пример5. Выполнить действия: а) 2i 3i; b) (2-3i)-(2+3i); с) (5-4i)-(3+2i).

а) 2i 3i=6i2=-6;

b) (2-3i)-(2+3i)=4-9i2=4+9=13;

с) (5-4i)-(3+2i)=(53-(-4)2)+ i(5+3(-4)=23-2i.

Можно выполнить умножение по правилу умножения многочленов:

(5-4i)-(3+2i)=15+10i -12 i +8=23-2i.

4) Деление комплексных чисел определяется как операция, обратная умножению, то есть число называется частным от деления z1 на z2, если . Тогда

.

Окончательно .

B тригонометрической форме:

.


Операция деления возможна только в случае, когда ).

Пример 6. Выполнить действия: а) ; b) ; с) ; d)

и указать вещественную и мнимую части полученного комплексного числа.

Решение.

а)Умножаем делимое и делитель на i, получим .


b) Умножаем делимое и делитель на множитель, сопряженный делителю: .


с) Умножаем делимое и делитель на множитель, сопряженный делителю: .


d) Умножаем делимое и делитель на множитель, сопряженный делителю:

;

Вещественная и мнимая части равны: , .

5) Возведение в степень и извлечение корней. Если комплексное число задано тригонометрической формой , то справедлива формула Муавра

. (1.4)

6) Для извлечения корня n-й степени (n – целое число, большее 1) из комплексного числа, заданного в тригонометрической форме, применяется формула, дающая n значений этого корня:

, k=0,1,…,n-1. (1.5)

Пример 7. Вычислить: .

Решение. Чтобы воспользоваться формулой Муавра, необходимо представить комплексное число в тригонометрической форме.

Имеем: ; и , т.е. (так как соответствующая точка лежит во второй четверти). Следовательно, и (в силу (1.4)). Учитывая, что и используя свойства тригонометрических функций, получаем:

.

Пример 8. Возвести число в пятую степень.

Решение. Получим тригонометрическую форму записи числа z. . Отсюда , а . Тогда по формуле Муавра получим:

.

Пример 9. Вычислить: .

Тригонометрическая форма заданного числа имеет вид (|z|=1), поэтому в силу (1.5)

, k=0,1,2.

Выписываем три искомых корня:

;

;

.


Практическая часть.

Задание 1. Выполните сложение комплексных чисел, выпишите вещественную и мнимую части полученных комплексных чисел:

а) (5+3i)+(1+10i); б) (3+i)+(-3-8i); в) (-6+2i)+(-6-2i).

Задание 2. Выполните действия:

а) (2-3i)+(5+6i)+(-3-4i); б) (1-i)-(7-3i)-(2+i)+(6-2i).

Задание 3. Выполните умножение комплексных чисел:

а) (5-3i)2i ; б) -i в) (5+3i)(2-5i); г)(3+4i)(3-4i).

Задание 4. Выполните деление комплексных чисел:

а) ; б) ; в) ; г) .

Задание 5. Запишите комплексные числа в тригонометрической форме:

а) 3i ; б) ; в) 2-2i; г) -i

Задание 6. Решите уравнения:

а) ; г) ;

б) ; д) ;

в) ; е) .

Задание 7. Выполните действия:

а)(1-i)12; б) ; в); г)





Практическая работа № 3.

Основное логарифмическое тождество. Правила действий с логарифмами.

Цель занятия: научить решать задачи, на применение основного логарифмического тождества для вычисления логарифмических выражений.

Учебная. Повторить определение логарифма числа, основное логарифмическое тождество. Показать применение логарифмов к решению логарифмических выражений.

Воспитательная: воспитание нравственного поведения,расширение кругозора.

Вопросы:


  1. Дать определение логарифмического тождества


  1. дать определение логарифма (показатель степени. ..)


  1. Чему равен логарифм произведения двух чисел? Привести пример


  1. Чему равен логарифм частного двух чисел? Привести пример


  1. Назовите основное логарифмическое тождество. Привести пример

  2. Вычислить следующие выражения.


Образец решения


10lg5x = 15 --> 5x = 15(:5) x =3 Ответ х = 3

1.Выполнить действия:


  1. а18 b29 c27 a23 b13 : a25b27 c25


2)

10

lg 4 x

18;

3)

12

log1 2 5








4) 7log 7

9 ;


5)

25

log5

x


6)

15log1 5 4 x28;

7) 100 lg 5



8) 49log 7

9 ;


5)

125

log5

x 12





Практическая работа








1 вариант


1.

Прологарифмируйте по основанию 10

выражение 7a3 3 b2

2.

Вычислить:

a) log49 84 – log4 9 12; b) log 72 18

+ log 72 4

3.Найдите х, если log x = 2log5 3 + ½ log5 49 – 1/3 log527

4)

log5 (x2 -

10) = log 5 9x;



hello_html_m442ac670.jpg


  1. вариант



  1. Прологарифмируйте по основанию 10 выражение a53 hello_html_m10ec15e9.jpgb4

hello_html_4f673c3b.jpg


Вычислить a)

log 36 84 – log36 14; b) log 2 192 - log2 3

3.Найдите х. если

log 7 x =2log 7 5 + ½ log 7 36 – 1/3 log7 125

4)

log 7 (x2 + 6x)=1;


  1. вариант



  1. Прологарифмируйте по основанию 10 выражение 8 a5 3 hello_html_m10ec15e9.jpgb5

hello_html_4f673c3b.jpg


2. Вычислить a) log 6 72 – log 6 2; b) log 27 243- log27 9


    1. 3. Найдите х. если log8 x = log 8 5 + ½ log 8 121 – 1/3 log 8125


4.log 6 (x2 - 5x)=1;


  1. вариант



  1. Прологарифмируйте по основанию 10 выражение 100 a3 3 hello_html_m10ec15e9.jpgb10

hello_html_4f673c3b.jpg


2. Вычислить a)

log 18 252 – log18

14; b)

log 20 40 +

log20 10



3.Найдитех. если

log 6 x = log 6 5 + ½ log 6

81 – 1/3 log 6 343



4. log5 (x2 + 10) = log 5 14;

































.


Ответ


Ответ


Ответ

Ответ


Вариант

1 задания

2 задания


3 задания

4


а








задания




















1.

lg7 + 3lg a + 2|3 lg|b|

a) 0,5;


b) 1;


24;

9











2.

4 + 6lg |a| + 0,6 lgb;

0,5;


6


30

- 7











3.

lg 8 + 5lga + 5|3 lg|b|

2


1


11

6






















4. 2 + 3lg|a| 10|3 lg|b| 1 2 45/7 2; - 2



Литература основная : Алимов Ш.А. «Алгебра и начала анализа» 10-11 класс с. 17-18


Дополнительная: Башмаков М.И. «Математика» М.: Высшая школа - 1994 , 542 с


Практическая работа № 4.

Преобразование алгебраических выражений различных типов.


Цель

занятия

учебная

Проверить знания и практические умения студентов по преобразованию алгебраических, рациональных, иррациональных, степенных выражений.

воспитательная и

развивающая

Способствовать овладению необходимыми навыками самостоятельной учебной деятельности; содействовать развитию умений применять полученные знания в типовых условиях

Межпредметные

связи

обеспечивающие

Математика (школьный курс)

обеспечиваемые

Физика, химия, техническая механика, экономика, курсовое и дипломное проектирование


Обеспечение урока:

Использование ИКТ (информационно – коммуникационных технологий)

(мультимедийные презентации, проекционное оборудование, интерактивная доска, персональный компьютер, компьютерное тестирование)

http://free.megacampus.ru/xbookM0001/index.html?go=part-006*page.htm

Цель работы: выполнить действия по преобразованию алгебраических, рациональных, иррациональных, степенных выражений.

Теоретические сведения:

КОРНИ НАТУРАЛЬНОЙ СТЕПЕНИ ИЗ ЧИСЛА, ИХ СВОЙСТВА.

Корень n – степени: , n - показатель корня, а – подкоренное выражение

Если n – нечетное число, то выражение имеет смысл при а

Если n – четное число, то выражение имеет смысл при

Арифметический корень:

Корень нечетной степени из отрицательного числа:

ОСНОВНЫЕ СВОЙСТВА КОРНЕЙ

  1. Правило извлечения корня из произведения:





  1. Правило извлечения корня из дроби:




  1. Правило извлечения корня из корня:





  1. Правило вынесения множителя из под знака корня:





  1. Внесение множителя под знак корня:

,



  1. Показатель корня и показатель подкоренного выражения можно умножить на одно и тоже число.



  1. Правило возведения корня в степень.



СТЕПЕНЬ С НАТУРАЛЬНЫМ ПОКАЗАТЕЛЕМ

=hello_html_1e91a43d.png,a – основание степени, n – показатель степени



Свойства:

  1. При умножении степеней с одинаковыми основаниями показатели складываются, а основание остается неизменным. 



  1. При делении степеней с одинаковыми основаниями показатели вычитаются, а основание остается неизменным. 



  1. При возведении степени в степень показатели перемножаются. 



  1. При возведении в степень произведения двух чисел, каждое число возводят в эту степень, а результаты перемножают.



  1. Если в степень возводят частное двух чисел, то в эту степень возводят числитель и знаменатель, а результат делят друг на друга.



  1. Если



СТЕПЕНЬ С ЦЕЛЫМ ПОКАЗАТЕЛЕМ



  1. По определению:





Свойства:

  1. Пусть r рациональное число , тогда

при r>0 > при r<0

7 .Для любого рациональных чисел r и s из неравенства > следует

> при a>1 при




Формулы сокращённого умножения.

hello_html_312ceb8f.png

Пример 1. Упростите выражение hello_html_34dc3388.png.

Решение

Применим свойства степеней (умножение степеней с одинаковым основанием и деление степеней с одинаковым основанием): hello_html_ma8781c3.png.

Ответ: 9m7 .

Пример 2.Сократить дробь: hello_html_m6398e2af.png

Решение. Так область определения дроби hello_html_m6398e2af.png все числа, кроме х ≠ 1 и х ≠ -2.Вместе с тем hello_html_m75d6f951.png.Сократив дробь, получим hello_html_34f6e61d.png.Область определения полученной дроби: х ≠ -2, т.е. шире, чем область определения первоначальной дроби. Поэтому дроби hello_html_m6398e2af.png и hello_html_34f6e61d.png равны при х ≠ 1 и х ≠ -2.

Пример 3.Сократить дробь: hello_html_304e84c2.png

Пример 4.Упростить: hello_html_m122a66f0.png

Пример 5.Упростить: hello_html_5fa13b83.png

Пример 6. Упростить: hello_html_m7fbd0e68.png

Пример 7. Упростить: hello_html_m414ff7a6.png

Пример 8.Упростить: hello_html_3e23c1c1.png

Пример 9. Вычислить: hello_html_m61f83d47.png.

Решение.hello_html_584dfff6.png

Пример 10.Упростить выражение: hello_html_3914b54a.png

Решение.hello_html_m244fc07f.png

Пример 11.Сократить дробь hello_html_m707c1b23.png, если hello_html_m2113757b.png

Решение.hello_html_2417f58b.png.

Пример 12.Освободиться от иррациональности в знаменателе дроби hello_html_m124b7021.png

Решение.В знаменателе имеем иррациональность 2-й степени, поэтому помножим и числитель, и знаменатель дроби на сопряженное выражение, то есть сумму чисел hello_html_m259685d.png и hello_html_3411f76d.png, тогда в знаменателе будем иметь разность квадратов, которая и ликвидирует иррациональность.hello_html_2c410fb9.png


ВАРИАНТ - I

1. Упростите выражение:


2. Найдите значение выражения:


3. Представьте степень с дробным показателем в виде корня


4. Привести указанное выражение к виду , где а -рациональное число, b – натуральное число

,

5. Упростить:

;

6. Замените арифметические корни степенями с дробным показателем

, ,

7. Представьте выражение в виде дроби, знаменатель которой не содержит знака корня

10. Выполните действие:

8. Сократите дробь

9. Выполните действие



ВАРИАНТ - II

1. Упростите выражение:


2. Найдите значение выражения:


3. Представьте степень с дробным показателем в виде корня


4. Привести указанное выражение к виду , где а- рациональное число, b – натуральное число

,

5. Упростить:

;

6. Замените арифметические корни степенями с дробным показателем

, ,

7. Представьте выражение в виде дроби, знаменатель которой не содержит знака корня

10. Выполните действие:

8. Сократите дробь

9. Выполните действие



ВАРИАНТ - III


1. Выполните действие:


2. Найдите значение выражения:


3. Представьте степень с дробным показателем в виде корня


4. Привести указанное выражение к виду , где а -рациональное число, b – натуральное число

,

5. Упростить:

;

6. Замените арифметические корни степенями с дробным показателем

, ,

7. Представьте выражение в виде дроби, знаменатель которой не содержит знака корня

10. Выполните действие:

8. Сократите дробь


9. Выполните действие


ВАРИАНТ - IV

1. Выполните действие:


2. Найдите значение выражения:


3. Представьте степень с дробным показателем в виде корня

,

4. Привести указанное выражение к виду , где а- рациональное число, b – натуральное число

,

5. Упростить:

;

6. Замените арифметические корни степенями с дробным показателем

, ,

7. Представьте выражение в виде дроби, знаменатель которой не содержит знака корня

10. Выполните действие:

8. Сократите дробь


9. Выполните действие



ВАРИАНТ - V

1. Упростите выражение:



2. Найдите значение выражения:


3. Представьте степень с дробным показателем в виде корня

,

4. Привести указанное выражение к виду , где а- рациональное число, b – натуральное число

,

5. Упростить:

;

6. Замените арифметические корни степенями с дробным показателем

, ,

7. Представьте выражение в виде дроби, знаменатель которой не содержит знака корня

10. Выполните действие:

8. Сократите дробь


9. Выполните действие


ВАРИАНТ - VI

1. Упростите выражение:


2. Найдите значение выражения:


3. Представьте степень с дробным показателем в виде корня

,

4. Привести указанное выражение к виду , где -а рациональное число, b – натуральное число

,

5. Упростить:

;

6. Замените арифметические корни степенями с дробным показателем

, ,

7. Представьте выражение в виде дроби, знаменатель которой не содержит знака корня

10. Выполните действие

8. Сократите дробь


9. Выполните действие



Практическая работа № 5.

Решение тригонометрических уравнений.


ЦЕЛЬ РАБОТЫ: закрепить навыки определения типов тригонометрических уравнений (простейшее, квадратное относительно sin x, cos x, tgx , однородное относительно sin x и cos x , уравнение, решаемое разложением на множители левой части), усвоить алгоритмы решения основных типов тригонометрических уравнений.

ПОРЯДОК ВЫПОЛНЕНИЯ РАБОТЫ:


  1. Ответить на контрольные вопросы:


а) Дайте определения арксинуса, арккосинуса арктангенса и арккотангенса числа а.

б)Перечислите свойства обратных тригонометрических функций.

в) Вспомните формулы, с помощью которых решают простейшие тригонометрические уравнения.

уравнение?

Объясните алгоритм его решения.


д) Какой вид имеет однородное относительно sin x и cos x тригонометрическое уравнение? Какова методика его решения?

е) Вспомните формулы, с помощью которых решают простейшие тригонометрические уравнения.

  1. По образцу выполнить тренировочные задания.


  1. Изучить условие задания для самостоятельной работы.


  1. Оформить отчет о работе.

Опр.


Уравнение называется тригонометрическим, если неизвестная величина входит в него как аргумент тригонометрической функции.


hello_html_m176964d2.jpg

















































































Уравнения вида корней:

x


cos x a

x




















sin x = a , cos x = a , tg x = a называются простейшими. Для них выведены формулы 1n arcsin a n, n Z

  • arccos a 2n, n Z


tg x a


x arctg a n, n Z



ctg x a



1

n, n Z


a





hello_html_m5ee105cc.jpg


  • этим уравнениям сводятся все другие. Для большинства таких уравнений требуется применение различных формул и преобразование тригонометрических выражений.


1. Уравнения, сводящиеся к квадратным 8sin 2 x 6sin x 3 0 . Вводят новую переменную sin x= thello_html_1de3c7c1.gif

2. Уравнения вида a sin x b cos x 0 а ≠ 0, b ≠ 0 называются однородными относительно sin x и


cos x. Оно решается делением обеих частей на cos x 0 . В результате получается уравнение

a tgx b 0 . Этим же способом решается уравнение 2 sin2 x 5 sin x · cos x + 3 cos 2 x = 0 . Обе


части



уравнения делятся на cos 2 x



или

hello_html_63a487a7.jpg



sin 2 x .








































  1. Уравнения, решаемые разложением левой части на множители Пример


sin 2x sin x 0


2 sin x cos x sin x 0 Общий множитель sin x выносится за скобки.

sin x 2 cos x 1 0


sin x 0 x n, n




Ответ:


или 2 cos x 1 0


  • Zcos x 1 2


  1. 2n, n Z 3

hello_html_5da47bc5.jpg

x

2n,

n Z




3







hello_html_m34ad9fc0.jpg



















Если уравнение имеет две серии корней, полученных при решении тригонометрических уравнений,


имеющую общую часть, в ответе можно оставлять обе серии. Например, х = πn ; x =



Литература:

1. Ш.А.Алимов «Алгебра и начала анализа» 10-11 кл. , стр. 165-190



  • n , n Z 3



Практическая работа

Тема: Решение тригонометрических уравнений.

Цель: Применение знаний к решению задач.

1 вариант

Решить уравнения:



1)

2 sin x


12ctgx 1 0





6






3)

2sin 2x 3cos 2x


5)

sin 5x sin x






2)


4)


6)




tgx 9ctgx 10 0




3sin

2

x sin x

cos x 2 cos

2

x 0





sin 4x sin

2

2x 0










______________________________________________________________________________________


Практическая работа

Тема: Решение тригонометрических уравнений.

Цель: Применение знаний к решению задач.

  1. вариант



x





2










1)

1 2 cos


1 3ctgx 0

2)

4sin


x cos x 1 0






4






3)

4sin x cos x 0

4)

3sin 2 x 7sin x cos x 2 cos 2 x 0


5)

cos 3x cos 5x sin 4x

6)

2sin x cos x cos x


hello_html_m6336215e.jpghello_html_50a5b7ee.jpg

УКАЗАНИЯ К ВЫПОЛНЕНИЮ ПРАКТИЧЕСКОЙ РАБОТЫ:


ПРИМЕР 1. Решите уравнение: 2 sin 2 x 5 cos x 1 0 .


РЕШЕНИЕ. Применив основное тригонометрическое тождество:


sin2 x 1 cos2 x , получим:


21 cos2 x 5 cos x 1 0 ,


2 2 cos 2 x 5 cos x 1 0 ,


2 cos 2 x 5 cos x 3 0 .


Это уравнение является квадратным относительно cos x . Обозначим cos x y , тогда 2 y 2 5 y 3 0 . Полученное уравнение имеет решения


y


1

.




2





1


2












Составим два простейших уравнения:









cos x 3

и


cos x

1

.



2











Первое уравнение решений не имеет, так как 1 cos x 1 . Второе уравнение имеет решение:


x arccos 12 2n ,



x 3 2n .


Ответ: x

2n, n Z










3





















ВАРИАНТЫ ПРАКТИЧЕСКОЙ РАБОТЫ.


Вариант 1

Выясните, к какому типу относятся данные тригонометрические уравнения, и решите их:


  1. sin2 x 2 sin x 3 0 ;

  2. 7 sin 2 x 8 sin x cos x 15 cos2 x ;

  3. cos 2x cos x ;

  4. sin 3x cos 2x sin 5x .

Вариант 2

Выясните, к какому типу относятся данные тригонометрические уравнения, и решите их:

  1. 2 sin 2 x 5 cos x 1 0 ;

  2. sin2 x 5 sin x cos x 6 cos 2 x 0 ;

  3. 7 sin x 3 cos 2x 0 ;

  4. 4 sin 2x cos 2x 1 0 .

Вариант 3

Выясните, к какому типу относятся данные тригонометрические уравнения, и решите их:

  1. tg 2 x 2tgx 3 0 ;

  2. 2 sin2 x 5 sin x cos x 3cos2 x 0 ;

  3. sin 2x 2 sin2 x ;

  4. cos x cos 3x cos 5x cos 7x .

Вариант 4

Выясните, к какому типу относятся данные тригонометрические уравнения, и решите их:

  1. 2 cos 2 x 3cos x 2 0 ;

  2. 3cos2 x 4 sin x cos x sin2 x ;


  1. sin2 x cos2 x sin 2x ;

  2. cos x cos 5x 0,5 cos 4x .

Вариант 5

Выясните, к какому типу относятся данные тригонометрические уравнения, и решите их:

  1. 5tg 2 x 13tgx 6 0 ;


  1. 3 sin2 x 7 sin x cos x 2 cos2 x 0 ;

  2. cos 2x cos x 0 ;

  3. cos 4x cos 2x cos 5x cos x .

Вариант 6

Выясните, к какому типу относятся данные тригонометрические уравнения, и решите их:

  1. 2 sin2 x 3cos x ;

  2. sin2 x 1,5 cos2 x 2,5 sin x cos x ;

  3. sin 2x 2hello_html_624902a7.jpghello_html_6a0a91c2.jpg3 sin 2 x 0 ;

  4. cos 3x cos x 0 .


Вариант 7

Выясните, к какому типу относятся данные тригонометрические уравнения, и решите их:

  1. 2 cos 2 x 4 sin 2 x 3 ;

  2. sin2 x 2 sin x cos x 3cos 2 x 0 ;

  3. cos 2x cos x ;

  4. cos 2x cos 3x sin 6x sin x .

Вариант 8

Выясните, к какому типу относятся данные тригонометрические уравнения, и решите их:

1. tg 2 x 4tgx 3 0 ;

  1. 3 sin2 x 4 sin x cos x cos 2 x 0 ;

  2. cos 2x 2 sin2 x ;


  1. sin 6x cos 2x sin 5x cos 3x .

Вариант 9

Выясните, к какому типу относятся данные тригонометрические уравнения, и решите их:

  1. 3 sin 2 x cos 2 x 0 ;

  2. sin2 x sin x cos x 2 cos2 x 0 ;

  3. sin 2x cos x ;

  4. cos 3x cos x sin 3x sin x .

Вариант 10

Выясните, к какому типу относятся данные тригонометрические уравнения, и решите их:

  1. 2 sin 2 x 3 sin x 2 ;

  2. 2 sin2 x sin x cos x cos2 x 0 ;


  1. sin 2x 2 cos2 x ;


  1. сos4x cos x 0 .




Практическая работа № 6.

Решение тригонометрических неравенств.


Цель: Применение знаний при решении задач.


Методические рекомендации

Опр.


Неравенства, содержащие переменную под знаком тригонометрической функции, называются тригонометрическими.


При решении тригонометрических неравенств используют единичную окружность.



Задача Решить неравенство cos x >

hello_html_6b53c179.jpg

1 2

hello_html_m317a79f6.jpg














По определению cos x – это абсцисса точки единичной окружности. Абсциссу, равную

1

, имеют


2







две точки единичной окружности М1 и М2 . Абсциссу, большую

1

имеют все точки М дуги


2







единичной окружности, лежащие правее прямой М1М2 . Таким образом, решениями неравенстваhello_html_m317a79f6.jpghello_html_m317a79f6.jpg

cos x >

1

являются все числа х из промежутка

x

.






2

3

3



















Все решения данного неравенства – множество интервалов


2n x

2n, n Z



3

3


















Ответ:

2n x

2n, n Z











3

3

























hello_html_m317a79f6.jpghello_html_md40e919.jpghello_html_103634c9.gif



























Литература:

  1. Ш.А.Алимов «Алгебра и начала анализа» 10-11 кл. , стр. 191-193


hello_html_4876f7bb.jpghello_html_4d8f6ba8.jpghello_html_5da47bc5.jpghello_html_m5956485e.jpg



Задания:

Вариант 1.

Решите неравенства:

а)

б)

Вариант 2.

Решите неравенства:

а)

б)

Вариант 3.

Решите неравенства:

а)

б)

Вариант 4.

Решите неравенства:

а)

б)

Решите тригонометрические неравенства:

1) ; 2) cos x 3) sin x 4) 5) ; 6) cos x

7) sin x 8) 9) 10) 11) 12)

13) 14) 15) 16) 17) ; 18) cos x 19)sin x 20) 21) ; 22) cos x 23) sin x 24) 25) 26) 27) 28) 29) 30) 31) 32)




Практическая работа № 7.

Решение тригонометрических уравнений и неравенств.


Вариант 1.


  1. Решите уравнение:

а) ;

б) .

  1. Решите уравнение:

а) ;

б) .

  1. Решите уравнение:

.

  1. Решите неравенство:


  1. Решите уравнение:

а) ;

б) .


  1. Решите уравнение:

.


Вариант 2.


  1. Решите уравнение:

а) ; б) .

  1. Решите уравнение:

а) ; б) .


  1. Решите уравнение:

.


  1. Решите неравенство:


  1. Решите уравнение:

а) ;

б) .


  1. Решите уравнение:

.





Практическая работа № 8.

Построение графиков функций, заданных различными способами.


Цель: Закрепление навыков построения графиков функций элементарными методами, формирование умения строить графики функций с помощью основных операций над графиками функций, воспитание настойчивости для достижения конечного результата, развитие навыков самоконтроля, взаимопомощи.

Вы уже знаете, как строить графики функций с помощью производной, преобразований графиков функций, с помощью асимптот. Сегодня, вы узнаете ещё один способ построения графиков сложных функций на примере функций вида:

f(kx+b) и f(ax2+bx+c).

Рассмотрим алгоритм построения графиков таких функций:

а) Найти область определения функции y=f (k x+b) или y=f (a x2+b x+c);

b) Разбить функцию на две: z (x)= k x+b или g (x)= a x2+b x+c

и y=f (z) или y=f (g);

с) Построить график функции z (x)= k x+b или g (x)= a x2+b x+c

и отметить особые точки (точки пересечения с осями координат, промежуточные точки);

d) Произвести заданные операции над ординатами выбранных точек, то есть вычислить значения y=f (zn) или y=f (gn);

e) Нанести полученные точки на рисунок, так чтобы ось z и ось y лежали на одной прямой, соединить отмеченные точки плавной линией.


Разберём этот алгоритм на примере функции y= (2-).

1. D(y)=R, z (x)= , y (z)=z3 (x).

2. Построим график функции z (x)= , отметим точки пересечения с осями координат, А (0;2) и С (6;0) и ещё две промежуточные точки В (3;1) и D (12;-2).

3. Вычислим ординаты этих точек:

z1=23=8 z2= 03=0 z3=13=1 z4=(-2)3=-8

4. Наносим новые точки на рисунок

А1 (0;8) В1 (3;1) С1 (6;0) D1 (12;-8).

5. Плавной линией соединяем полученные точки.










Задания по группам:




  1. Рассмотрим построение графика функции вида y=f (a x2+b x+c).

а) Повторить алгоритм построения графика квадратичной функции;

b) Построить график функции: y =(x2-4x+3) 2

1.D (y)=R, g(x)= x2-4x+3 y(x)=g2(x)

Строим параболу по точкам: А(0;3), В(1;0), С(2;-1), D(3;0), E(4;3).

Найдём ординаты этих точек.

g1=32=9, g2=02=0, g3=(-1)2=1, g4=02=0, g5=32=9.

Наносим новые точки на рисунок, плавной линией соединяем полученные точки.

  1. Групповая работа. На миллиметровой бумаге построить графики функций:










Какими способами можно построить графики функций?

По какому алгоритму можно построить графики функций вида y=f (k x+b) и y=f (a x2+b x+c).


Приложения.


Построить график функции

=







Построить график функции

1 D (у)=(-∞;-4)

2 Прямые х=2 и х=-4- вертикальные асимптоты

3 Промежутки знакопостоянства




4 , значит у =0-горизонтальная асимптота.

С учетом этого построим график функции асимптотическим методом














Построить график функции






Построить график функции

При х>0, у =1

При х<0, у =






Построить график функции

При

При






Практическая работа № 9.

Преобразование графиков.



Цель: Постройте графики функций, используя различные преобразования, ответьте на вопрос задачи.

Выполнение работы

Методические указания

Работа рассчитана на 10 вариантов, номер варианта совпадает с последней цифрой порядкового номере в списке. Например, 1, 11, 21, 31 …выполняют 1 вариант, 2,12, 22 … - 2 вариант, и т.д.

Работа состоит из двух частей: первая часть задания 1 – 5, это задания которые обязательно нужно выполнить, чтобы получить зачет, если эти задания выполнены с ошибкой, необходимо их исправить и снова сдать работу на проверку. Вторая часть, содержит задания, выполнив которые, вы можете заработать дополнительную оценку: основная часть +2 задания – «4», основная часть +3 задания – «5».

Задание 1. Графиком линейной функции является прямая, для ее построения достаточно двух точек. (значения аргумента х берем произвольно, а значение функции у, считаем подставляя в формулу).

Чтобы проверить проходит ли график функции через указанную точку нужно координаты точки подставить вместо х и у, если получили верное равенство, то прямая проходит через указанную точку, в противном случае – не проходит.

Задание 2, 3, 4. Графики указанных функций получаются из графиков функций , используя сдвиг вдоль оси х или у.

, сначала строим график функции или , затем сдвигаем его на «а» единиц вправо или влево (+а – влево, - а вправо), затем сдвигаем на «в» единиц вверх или вниз (+в – вверх, -в – вниз)

Аналогично с другими функциями:

Задание 5 Чтобы построить график функции: , нужно: 1) построить график функции , 2) часть графика которая находится выше оси х оставить без изменения, 3) часть графика, которая находится ниже оси х зеркально отобразить.

Задачи для самостоятельного решения.

Обязательная часть

Задание 1. Постройте график линейной функции, определите, проходит ли график функции через указанную точку:

  1. , А(42 ;26)

  2. , В(42;19)

  3. . С(-33;6)

  4. D(-40;77)

  5. , M(20;64)

  6. E(-20;8)

  7. ,F(60;18)

  8. , K(-30;86)

  9. , Z(-21;-47)

  10. , N(-50;-22)


Задание 2. Постройте график квадратичной функции, укажите множество значений данной функции.

  1. Задание 3. Постройте график функции, определите, возрастает или убывает указанная функция.

  1. Задание 4. Постройте график функции, ответьте на вопрос задачи.

  1. , укажите наименьшее значение функции.

  2. , укажите наименьшее значение функции.

  3. , укажите наименьшее значение функции.

  4. , укажите наименьшее значение функции.

  5. , укажите наибольшее значение функции.

  6. , укажите наибольшее значение функции.

  7. , укажите наибольшее значение функции.

  8. , укажите наибольшее значение функции.

  9. , укажите наименьшее значение функции.

  10. , укажите наибольшее значение функции.

  1. Задание 5. Постройте график функции, содержащей знак модуля.

  1. Задачи на дополнительную оценку.

  2. Задание 6. Постройте график функции, заданной кусочно, определите, есть ли точка разрыва у данной функции:

  1. Задание 7. Определите, сколько решений имеет система уравнений, отвеет обоснуйте.

  1. Задание 8. Постройте график по описанию.

  1. Область определения: ; Множество значений: ; Точки пересечения с осью Х: (-2;0), (3;0), (7;0); Точка пересечения с осью У (0;-3); Точки максимума: (-5;5) и (5;2); Точка минимума: (1;-4); Дополнительные точки: (-7;3) и (9;-6).

  2. Область определения: ; Множество значений: ; Точки пересечения с осью Х: (5;0), (9;0), Точка пересечения с осью У (0;4); Точка максимума: (3;5); Точки минимума: (1;3) (7;-3); Дополнительные точки: (-2;7) и (10;3)

  3. Область определения: ; Множество значений: ; Точки пересечения с осью Х: (-1;0), (4;0), (7;0); Точка пересечения с осью У (0;-1,5); Точки максимума: (-3;4) и (6;5); Точка минимума: (1;-2); Дополнительные точки: (-4;2) и (8;-4)

  4. Область определения: ; Множество значений: ; Точки пересечения с осью Х: (-9;0), (-5;0) (-2;0), (1;0)Точка пересечения с осью У (0;3); Точки максимума: (-7;3); (-1;6) Точки минимума: (-3-6); Дополнительные точки: (-10;-2) и (4;-6).

  5. Область определения: ; Множество значений: ; Точки пересечения с осью Х: (5;0), (9;0) Точка пересечения с осью У (0;6); Точка максимума: (2;7); Точки минимума: (-3;3); (7;-6); Дополнительные точки: (-6;8) и (10;2).

  6. Область определения: ; Множество значений: ; Точки пересечения с осью Х: (-1;0), (2;0), (7;0) Точка пересечения с осью У (0;-1); Точки максимума: (-5;6); (5;7) Точки минимума: (1;-2); (8;-5); Дополнительные точки: (-8;3) и (10;-2).

  7. Область определения: ; Множество значений: ; Точки пересечения с осью Х: (7;0), (12;0) Точка пересечения с осью У (0;2); Точка максимума: (4;6); Точки минимума: (0;2); (9;-6); Дополнительные точки: (-4;8) и (14;5).

  8. Область определения: ; Множество значений: ; Точки пересечения с осью Х: (6;0), Точка пересечения с осью У (0;-9); Точка максимума: (-4;-1); Точка минимума: (2;-10);

  1. Дополнительные точки: (-8;-5) и (8;5).

  1. Область определения: ; Множество значений: ; Точки пересечения с осью Х: (5;0), (9;0) Точка пересечения с осью У (0;6); Точка максимума: (2;7); Точки минимума: (-3;3); (7;-6); Дополнительные точки: (-6;8) и (10;2).

  2. Область определения: ; Множество значений: ; Точки пересечения с осью Х: (5;0), Точка пересечения с осью У (0;4); Точки максимума: (-4;8); (2;6) Точка минимума: (-1;3); Дополнительные точки:

  1. (-10;2) и (6;-4).

  2. Сделайте выводы, ответив на вопросы.

  1. Графики каких функций вы строили в данной работе?

  2. Как называется график линейной функции?

  3. Как называется график квадратичной функции?

  4. Какие преобразования графиков вы знаете?

  5. Как в системе координат располагается график четной функции? График нечетной функции?

  1. Практическая работа № 10.

  2. Вычисление пределов.

  3. Цель: формирование умений вычислять пределы последовательностей и функций,

  4. раскрывать в простейших случаях неопределенности.

  5. Методические рекомендации

  1. Пределы числовых последовательностей

  1. Числовые последовательности. Формула общего члена.

  2. Предел числовой последовательности. Сходящаяся и

  3. расходящаяся последовательности. Ограниченная

  4.  последовательность. Монотонная последовательность.

  5. Теорема Вейерштрасса. Основные свойства пределов.

  6. Некоторые замечательные пределы.

  7. Последовательности.  Рассмотрим ряд натуральных чисел:

  8.  

  9. 1,  2,  3, … ,  n –1,  n, … .

  10.  

  11. Если заменить каждое натуральное число  n  в этом ряду некоторым числом  un , следуя некоторому закону, то мы получим новый ряд чисел:          

  12.  

  13. u1 ,   u2 ,   u3 , …,   un 1 ,   un  , …,  кратко обозначаемый { un }  

  14.  

  15. и называемый числовой последовательностью. Величина  un называется общим членом последовательности. Обычно числовая последовательность задаётся некоторой формулой  un = f ( n ), позволяющей найти любой член последовательности по его номеру  n ; эта формула называется формулой общего члена. Заметим, что задать числовую последовательность формулой общего члена не всегда возможно; иногда последовательность задаётся путём описания её членов (см. ниже последний пример).

  16.  

  17. П р и м е р ы    числовых последовательностей:

  18.  

  19.                          1,  2,  3,  4,  5, …   ряд натуральных чисел ;

  20.  

  21.                          2,  4,  6,  8,  10, … ряд чётных чисел;

  22.  

  23.                          1.4,  1.41,  1.414,  1.4142, … числовая последовательность

  24.                                                                             приближённых  значений hello_html_7fa030f1.gif

  25.                                                                             с увеличивающейся точностью.

  26. В последнем примере невозможно дать формулу общего члена последовательности, тем не менее эта последовательность описана полностью.

  27. Предел числовой последовательности. Рассмотрим числовую последовательность, общий член которой приближается к некоторому числу  a  при увеличении порядкового номера  n. В этом случае говорят, что числовая последовательность имеет предел. Это понятие имеет более строгое определение.

  28. hello_html_16949ec9.gif

  29. Это определение означает, что  a  есть предел числовой последовательности, если её общий член неограниченно приближается к  a  при возрастании  n. Геометрически это значит, что для любого  hello_html_m3bcdfda7.gif> 0  можно найти такое число N,  что начиная с  n > N  все члены последовательности расположены внутри интервала ( a hello_html_m3bcdfda7.gifa hello_html_m3bcdfda7.gif). Последовательность, имеющая предел, называется сходящейся; в противном случае – расходящейся.

  30. Последовательность называется ограниченной, если существует такое число M, что | un  | hello_html_mb6b13c7.gifMдля всех  n . Возрастающая или убывающая последовательность называется монотонной.

  31. Теорема Вейерштрасса. Всякая монотонная и ограниченная последовательность имеет предел (эта теорема даётся в средней школе без доказательства). 

  32. Если члены последовательностей { un }, { vn }, { wn } удовлетворяют неравенствам

  33.  Основные свойства пределов.  Нижеприведенные свойства пределов справедливы не только для числовых последовательностей, но и для функций.Если { un } и { vn }   две сходящиеся последовательности, то:hello_html_28cbb386.gif hello_html_m12a2463.gif
    Замечательные пределыhello_html_m494b84b2.gif

  1. Пределы функций

  1.  

  2. Предел функции. Некоторые замечательные пределы.

  3. Бесконечно малая и бесконечно большая величины.

  4.  Конечный предел. Бесконечный предел.

  5. Понятие бесконечности.

  6.  

  7. Предел функции. Число L называется пределом функции  y = f ( x ) при  x, стремящемся к  a :

  8. hello_html_5d50a8ef.gif
    если для любого  hello_html_m3bcdfda7.gif> 0 найдётся такое положительное число hello_html_m626de232.gif= hello_html_m626de232.gif( hello_html_m3bcdfda7.gif), зависящее от  hello_html_m3bcdfda7.gif, что из условия | x - a | < hello_html_m626de232.gifследует  |  f ( x ) – L | < hello_html_m3bcdfda7.gif.

  9. Это определение означает, что L есть предел функции  y = f ( x ), если значение функции неограниченно приближается к  L , когда значение аргумента  x приближается к  a. Геометрически это значит, что для любого  hello_html_m3bcdfda7.gif> 0  можно найти такое число  hello_html_m626de232.gif, что если  x  находится в интервале ( a - hello_html_m626de232.gif, a + hello_html_m626de232.gif), то значение функции лежит в интервале ( L - hello_html_m3bcdfda7.gifL + hello_html_m3bcdfda7.gif). Отметим, что в соответствии с этим определением аргумент функции лишь приближается  к  a , не принимая этого значения! Это следует учитывать при вычислении предела любой функции в точке её разрыва, где функция не существует.

  10. П р и м е р .   Найти

  11.                                       hello_html_50daeb3c.gif

  12. Р е ш е н и е .  Подставляя  x = 3  в выражение  hello_html_14cb63eb.gifполучим не имеющее смысла                     выражение hello_html_10c699df.gif. Поэтому решим по-другому:

  13.                                                           hello_html_m7ecf838b.gif

  14. Сокращение дроби в данном случае корректно, так как  x hello_html_455e6af3.gif3 , он лишь приближается к 3.  Теперь мы имеем:

                                                                 
    hello_html_21608ec1.gif

  15.   поскольку, если  x  стремится к  3, то  x + 3  стремится к  6 .

  16. Замечательные пределы

  17.                                 hello_html_m4da4d87f.gif

  18. Бесконечно малая и бесконечно большая величины. Если предел некоторой переменной равен 0, то эта переменная называется бесконечно малой.

  19. П р и м е р .  Функция  y  =  hello_html_67cd7b95.gif  является бесконечно малой при  x,

  20.                       cтремящемся к  4, так как  hello_html_5ee0f046.gif

  21.  Если абсолютное значение некоторой переменной неограниченно возрастает, то эта переменная называется бесконечно большой.

  22. hello_html_m3e2b2102.gif
    Бесконечно большая величина не имеет
    конечного предела, но она имеет так называемый бесконечный предел, что записывается как:

  23.                                                   hello_html_m1d8cd51.gif

  24. Символ hello_html_3d922e10.gif ( “бесконечность” ) не означает некоторого числа, он означает только, что дробь неограниченно возрастает при  x, стремящемся к 3. Следует отметить, что дробь может быть как положительной ( при x > 3 ), так и отрицательной ( при x < 3 ). Если бесконечно большая величина может быть только положительной при любых значениях  x, это отражается в записи. Например, при  x hello_html_2c63fb57.gif 0 функция  y = x2 бесконечно большая, но она положительна как при  x > 0, так и при  x < 0 ; это выражается так:

  25. hello_html_26d9063a.gif

  26. Наоборот, функция  yx 2  всегда отрицательна, поэтому  

  27. hello_html_m4c3b4b44.gif

  28. В соответствии с этим, результат в нашем примере можно записать так:

  29. hello_html_m5c3a6505.gif

  30. hello_html_m2c3b0c07.gif

  31. Практическая работа по теме:

  32. «Вычисление пределов»

  33. Вариант 1.

  34. 1.Найдите пределы последовательностей:

  1. ; 3) ;

  2. ; 4) .

  1. 2.Найдите пределы функций:

  1. ;

  2. ;

  3. ;

  4. ;

  5. .

  1. 3. Раскрытие неопределенностей вида . Найдите пределы:

  1. ; 3) ;

  1. 2) ; 4) .

  2. 4. Раскрытие неопределенностей вида . Найдите пределы:

  3. 1) ; 2) .

  4. Практическая работа № 11.

  5. Нахождение производной функции.

  6. Цель: корректировать знания, умения и навыки в теме: «Вычисление производных функций», закрепить и систематизировать знания по теме.

  7. ПОРЯДОК ВЫПОЛНЕНИЯ РАБОТЫ:

  1. Ответить на контрольные вопросы:

  1. а) Какая функция называется сложной? Приведите примеры сложных функций.

  2. б) Сформулируйте правило вычисления производной сложной функции.

  1. По образцу выполнить тренировочные задания.

  2. Изучить условие заданий для практической работы.

  3. Оформить отчет о работе.

  1. Краткие теоретические и учебно-методические материалы по теме практического занятия

  2. Таблица производных основных элементарных функций:

    1.
  3. 2. hello_html_deaaee9.gif

  4. 3.

  5. 4.

  6. 5.

  7. 6.

  8. 7.

  9. 8.

  10. 9.

  11. 10.

  12. 11.

    1. 12.

    2. 13.

    3. 14.

    4. 15.

    5. 16.

    6. 17.

    7. 18.

    8. 19.

    9. 20.

  13. Вопросы для закрепления теоретического материала к практическому занятию:

  1. Дать определение производной.

  2. Записать формулы дифференцирования.

  1. Чему равна производная постоянной?

  2. УКАЗАНИЯ К ВЫПОЛНЕНИЮ ПРАКТИЧЕСКОЙ РАБОТЫ

  3. ПРИМЕР 1. Заданы функции hello_html_m170eb84f.gif. Задайте формулой сложную функцию h, если: а) hello_html_m7adb8bdd.gif; б) hello_html_m50c8678b.gif.

  4. РЕШЕНИЕ. а) Функцию h можно представить в виде сложной функции hello_html_m7adb8bdd.gif таким образом:

  5. hello_html_479386ad.gif.

  6. б) Функцию h можно представить в виде сложной функции hello_html_m50c8678b.gif таким образом:

  7. hello_html_m568e5863.gif.

  8. ПРИМЕР 2. Задайте формулами элементарные функции f и g, из которых составлена сложная функция hello_html_m7adb8bdd.gif: а) hello_html_44e9d1a2.gif; б) hello_html_m134f1ce2.gif.

  9. РЕШЕНИЕ. а) Функцию h можно представить в виде сложной функции hello_html_m7adb8bdd.gif, где

  10. hello_html_m49058b8f.gif.

  11. б) Функцию h можно представить в виде сложной функции hello_html_m7adb8bdd.gif, где hello_html_m563947a2.gif.

  12. ПРИМЕР 3. Найдите производные сложных функций: а) hello_html_4bd028b2.gif; б) hello_html_m2dafcf2a.gif.

  13. РЕШЕНИЕ. а) Так как hello_html_m269cc0c.gif, где hello_html_6a371f9d.gif, то hello_html_m68e2d9e9.gif и hello_html_m57a80bc8.gif, откуда hello_html_m160718b9.gif.

  14. б) Так как hello_html_m269cc0c.gif, где hello_html_45fb98c1.gif, то hello_html_m641b7826.gif и hello_html_3e6add21.gif, откуда hello_html_14c56fa0.gif.

  15. ЗАДАНИЯ ДЛЯ САМОПРОВЕРКИ.

  1. Задайте формулами элементарные функции f и g, из которых составлена сложная функция hello_html_m7adb8bdd.gif, если hello_html_1b5a94ad.gif.

  2. Найдите производную сложной функции hello_html_m277afec0.gif.

  1. Задания для практического занятия:

    1) ;
  2. 2) ;

  3. 3) ;

  4. 4) ;

  5. 5) ;

  6. 6) ;

  7. 7) ;

  8. 8) ;

    1. 1) ;

    2. 2) ;

    3. 3) ;

    4. 4) ;

    5. 5) ;

    6. 6) ;

    7. 7) ;

    8. 8) .

    1. 2.Вычислить производную функции, используя правила дифференцирования

    1. 1) ;

    2. 2) ;

    3. 3) ;

    1. 1) ;

    2. 2) ;

    3. 3) .

    1. 3.Решите уравнения f’ (х) = 0.

    1. f’ (х) = + – х2 – 3х;

    1. f’ (х) = + х3– – 2х

  9. Инструкция по выполнению практического занятия

  1. Познакомиться с конспектами лекций и краткой теоретической справкой

  2. Ответить устно на контрольные вопросы.

  3. Используя конспекты лекций, решить практические задания.

  1. ВАРИАНТЫ ПРАКТИЧЕСКОЙ РАБОТЫ

  2. Вариант 1.

  3. Вычислите производные сложных функций:

  4. а) hello_html_67975971.gif; б) hello_html_m2d1729dd.gif; в) hello_html_m18de1f6f.gif; г) hello_html_m59128d09.gif; д) hello_html_1ad0cb9b.gif.

  5. Вариант 2.

  6. Вычислите производные сложных функций:

  7. а) hello_html_md38745a.gif; б) hello_html_33252e17.gif; в) hello_html_m33a57541.gif; г) hello_html_m4410568f.gif;

  8. д) hello_html_m2aa49d59.gif.

  9. Вариант 3.

  10. Вычислите производные сложных функций:

  11. а) hello_html_27776a00.gif; б) hello_html_m52c71390.gif; в) hello_html_m55e57b29.gif; г) hello_html_44ed913b.gif;

  12. д) hello_html_m45ca79d4.gif.

  13. Вариант 4.

  14. Вычислите производные сложных функций:

  15. а) hello_html_m3ce10b53.gif; б) hello_html_764c01ea.gif; в) hello_html_3085db26.gif; г) hello_html_m74a567df.gif;

  16. д) hello_html_2437607d.gif.

  17. Вариант 5.

  18. Вычислите производные сложных функций:

  19. а) hello_html_3528686c.gif; б) hello_html_173c78bb.gif; в) hello_html_303dd031.gif;

  20. г) hello_html_2aa6030a.gif; д) hello_html_1a6b0355.gif.

  21. Вариант 6.

  22. Вычислите производные сложных функций:

  23. а) hello_html_30d65267.gif; б) hello_html_m306fa4b0.gif; в) hello_html_6f94e370.gif; г) hello_html_462c9a28.gif; д) hello_html_645fd6b6.gif.

  24. Вариант7.

  25. Вычислите производные сложных функций:

  26. а) hello_html_m5a7893ec.gif; б) hello_html_mbbd5023.gif; в) hello_html_415d2924.gif; г) hello_html_m74c164ac.gif;

  27. д) hello_html_m5d59723f.gif;

  28. Вариант 8.

  29. а) hello_html_30b36e4d.gif; б) hello_html_m50d93026.gif; в) hello_html_92e378f.gif; г) hello_html_bf6cc6b.gif; д) hello_html_469618a8.gif.

  30. Порядок выполнения отчёта по практической работе:

  31. 1. Выполнить задания.

  32. 2. Ответить на вопросы для закрепления теоретического материала (устно).

  33. 3. Оформить отчёт по практической работе.

  34. Образец отчёта по практической работе:

  35. Тема.

  36. Учебная цель.

  37. Название практической работы.

  38. Решение заданий практической работы.

  39. Ответы на вопросы для закрепления теоретического материала.

  40. Практическая работа №12.

  41. Использование производной для нахождения наилучшего решения в прикладных задачах.

  42. ЦЕЛЬ РАБОТЫ: корректировать знания, умения и навыки в теме: «Вычисление производных алгебраических функций», закрепить и систематизировать знания по теме, определить уровень усвоения знаний, оценить результат деятельности уч-ся.

  43. ПОРЯДОК ВЫПОЛНЕНИЯ РАБОТЫ:

  1. Ответить на контрольные вопросы:

  1. а) Сформулируйте определение функции.

  2. б) Сформулируйте правила вычисления производных алгебраических функций.

  3. в) В чем состоит механический смысл производной?

  4. г) Тело движется по прямой согласно закону х(t). Запишите формулы для нахождения скорости и ускорения тела в момент времени t.

  1. По образцу выполнить тренировочные задания.

  2. Изучить условие заданий для практической работы.

  3. Оформить отчет о работе.

  1. УКАЗАНИЯ К ВЫПОЛНЕНИЮ ПРАКТИЧЕСКОЙ РАБОТЫ

  2. f
    1. ПРИМЕР

    1. 1.

    1. Решите

    1. неравенство:

    1. x

    1. 0 ,

    1. если

    1. g x

    1. f x

    1. 1

    1. x 3

    1. 3x 2

    1. 5x, gx 2x 1,5x 2.

    1. 3

  3. РЕШЕНИЕ. Пользуясь правилами дифференцирования алгебраических

  4. функций и формулами дифференцирования элементарных функций,

  5. вычислим производные:

    1. 2

    1. 1

    1. 3

    1. 2

    1. 2

    1. f

    1. x

    1. 3x

    1. 5x

    1. x

    1. 3x

    1. 5x x 6x 5 ;

    1. x

    1. 3

    1. 3

    1. 2

    1. 2

    1. 2x 1,5x

    1. 2 3x .

    1. g x 2x 1,5x

  6. Таким образом, нужно решить неравенство:

  7. x 2 6x 5 0 .

  1. 3x

  1. Разложим числитель дроби на множители:

  2. x 2 6x 5 0, x 1, x
    1. 2

    1. 5; x 2 6x 5 x 1x 5.

    1. 1

    1. Неравенство

    1. x 1x 5

    1. 0 методом интервалов.

    1. 2 3x

    1. Нули числителя: х = 1, х = 5. Нуль знаменателя: x

    1. 3

    1. О т в е т:

    1. 2

    1. ;1

    1. 5;.

    1. 3

    1. ПРИМЕР 2. Тело движется по прямой согласно закону

    1. xt t 3 2t 5 .

    1. Найдите скорость и ускорение точки в момент времени t0 4 .

  3. hello_html_m1a7c760e.jpg

    1. 2

    1. 2t 5 3t 2 .

    1. vt x t t

    1. Значит, в момент времени t0 4

    1. скорость данного движения такова:

    1. v4 3 4 2 2 46 .

  4. РЕШЕНИЕ. Скорость движения – это производная от пути по времени, следовательно,

  5. Так как нам известна скорость движения как функция времени, мы можем

  6. найти ускорение этого движения:

  7. at vt 3t 2 2 6t .

  8. Значит, в момент времени t0 4 ускорение данного движения равно:

  9. a4 6 4 24 .

  10. О т в е т: 46; 24.

  11. ЗАДАНИЯ ДЛЯ САМОКОНТРОЛЯ.

    1. 1.

    1. Решите неравенство

    1. f x 0 , если f x 6x

    1. 7x

    1. 23 .

    1. 2.

    1. Тело движется по

    1. прямой согласно закону

    1. xt 3t 2 t 0,4 . Найдите

    1. скорость и ускорение точки в момент времени t0 2 .

  12. ВАРИАНТЫ ПРАКТИЧЕСКОЙ РАБОТЫ Вариант 1.

  1. Пользуясь формулами и правилами дифференцирования, найдите производные функций:

    1. x

    1. 2

    1. 1 x

    1. а) 5x 4 3,5x 2 x 6 ; б)

    1. x ; в)

    1. .

    1. 4 x 2

    1. x

    1. 2. Решите уравнение

    1. 2

    1. f x 0 , если f x 4

    1. x .

  1. hello_html_m7380bd0d.jpghello_html_m26f37992.jpg

  2. Вариант 2.

  1. Пользуясь формулами и правилами дифференцирования, найдите производные функций:

    1. x 3

    1. 3 ; б) x 2

    1. 3x 2

    1. ; в)

    1. 1 x 2

    1. .

    1. x

    1. x

    1. x

    1. 1 x3

    1. 3

    1. 2

    1. 2. Решите неравенство f x 0 , если

    1. f x 2x

    1. 9x

    1. 12x 5 .

  1. hello_html_m3280237b.jpghello_html_m7380bd0d.jpg

  2. Вариант 3.

  1. Пользуясь формулами и правилами дифференцирования, найдите производные функций:

,7x 5
  1. 2

  1. x 3

  1. 0,75x 2

  1. 1

  1. ; б) x 2sin x ; в)

  1. x 2

  1. .

  1. 3

  1. 10

  1. x

  1. 3

  1. 2. Решите уравнение

  1. 4

  1. 2

  1. f x 0 , если f x x 2x 1.

  1. Вариант 4.

  1. Пользуясь формулами и правилами дифференцирования, найдите производные функций:

x10 0,05x 4
  1. 1

  1. x 0,3 ; б) 4 x 2

  1. cos x ; в)

  1. sin x

  1. .

  1. 7

  1. 2 x 3

x 5
  1. 10x 3

  1. 2. Решите уравнение

  1. f x 0

  1. , если

  1. f x

  1. 9x .

  1. 5

  1. 3

  1. Вариант 5.

  1. Пользуясь формулами и правилами дифференцирования, найдите производные функций:

    1. x 4

    1. 1

    1. x 3

    1. 1

    1. x 2

    1. x 5 ; б) x 2 5 x ; в)

    1. x 3 3x

    1. .

    1. 4

    1. 2

    1. 3

    1. 1 2x

    1. 1

    1. 2. Решите уравнение

    1. f x 0

    1. , если f x 2x

    1. cos x .

  1. Вариант 6.

  1. Пользуясь формулами и правилами дифференцирования, найдите производные функций:

x
  1. 2x 2

  1. а) 2

  1. lg x 3

  1. ; б) 2

  1. x tgx ;

  1. в)

  1. .

  1. 3

  1. x

  1. 2. Материальная

  1. точка

  1. движется

  1. прямолинейно

  1. по

  1. закону

  1. xt

  1. 1

  1. t 3 2t 2 5t .

  1. Через сколько секунд после начала движения точка

  1. hello_html_m27bb263.jpg

  2. 3

  3. остановится?

  4. Вариант 7.

  1. Пользуясь формулами и правилами дифференцирования, найдите производные функций:

  1. ; б) x lg x ; в)

  1. x

  1. .

  1. x

  1. x 3

  1. 3 x

  1. 4 x

  1. 2. Найдите х,

  1. при котором

  1. f x

  1. 3 , если f x

  1. 1 x

  1. , gx

  1. 1

  1. .

  1. 2

  1. 2

  1. g x

  1. 4 x

  1. x

  1. 4

  1. hello_html_16385376.jpghello_html_16385376.jpg

  2. Вариант 8.

  1. Пользуясь формулами и правилами дифференцирования, найдите производные функций:

  1. 1

  1. x 3

  1. 4x 2 7x 18 ; б)

  1. ln x ; в)

  1. e x

  1. .

  1. x

  1. 3

  1. x

  1. 2. По прямой движутся две материальные точки по законам x t 4t 2

  1. 3 и

  1. x2 t t 3 .

  1. 1

  1. В каком промежутке времени скорость первой точки больше

  1. hello_html_69f7e159.jpg

  2. скорости второй?

  3. Практическая работа №13.

  4. Решение задач на отыскание наибольших и наименьших величин.

  5. ЦЕЛЬ РАБОТЫ:

  1. Корректировать знания, умения и навыки в теме: «Наибольшее и наименьшее значения функции на отрезке. Решение прикладных экстремальных задач».

  2. Закрепить и систематизировать знания по теме.

  3. Определить уровень усвоения знаний, оценить результат деятельности учащихся.

  1. ОБОРУДОВАНИЕ: инструкционно-технологические карты, таблицы производных элементарных функций, микрокалькуляторы.

  2. ПОРЯДОК ВЫПОЛНЕНИЯ РАБОТЫ:

  1. Ответить на контрольные вопросы:

  1. а) Какую точку называют критической точкой функции?

  2. б) Сформулируйте признак возрастания (убывания) функции.

  3. в) Сформулируйте признак максимума (минимума) функции.

  4. г) Опишите схему исследования функции.

  1. С помощью обучающих таблиц повторить планы нахождения наибольшего и наименьшего значений функции на отрезке, решения прикладных экстремальных задач и изучить образцы решенных примеров.

  2. Выполнить задания для самоконтроля (в таблице).

  3. Изучить условие заданий для практической работы.

  4. Оформить отчет о работе.

  1. Краткие теоретические сведения

  2. Наибольшее и наименьшее значение функции

  3. Наибольшим значением функции называется самое большее, а наименьшим - самое меньшее из всех ее значений.

  4. Функция может иметь только одно наибольшее значение и только одно наименьшее значение или может не иметь их совсем.

  5. Чтобы найти наибольшее и наименьшее значение функции на отрезке а ≤ х ≤ b, где она непрерывна, следует:

  1. Найти экстремумы функции на данном отрезке.

  2. Найти значение функции на концах отрезка: f(a) и f(b)

  3. Из всех найденных значений выбрать наибольшее и наименьшее.
    Пример 11. Найти наибольшее и наименьшее значение функции

  1. hello_html_m6b3a9105.jpg

  2. на отрезке [-2;4].

  3. Решение

  4. 1. Найдем экстремумы функции, для чего найдем производную функции и критические точки из условия у' =0 y' = 0 при x1=0, x 2 – 2x − 3 = 0 ,

  5. hello_html_72852e0.jpghello_html_m59a1bf17.jpg

  6. Отметим критические точки 1 рода x=−1, x=0, x=3, на числовой прямой (рис.7).

  7. Исследуем знак производной в каждом из полученных интервалов:

  8. hello_html_30dc4fd0.jpg

  9. у'(2) < 0, у'(−0,5) > 0, у'(1) < 0, у'(4) < 0.

  10. Таким образом,

  11. hello_html_44d7521c.jpg

  1. Наименьшее и наибольшее значения функции.

  1. Задание. Найдите наименьшее и наибольшее значения функции hello_html_m31890542.gif на промежутке hello_html_m432553e9.gif.

  2. шага

    1. План нахождения hello_html_m50ea1929.gif и hello_html_m1988140d.gif на hello_html_m41cdf12f.gif

    1. Применение плана

    1. 1

    1. Находим производную функции

    1. hello_html_178b7d8f.gif

    1. 2

    1. Находим критические точки функции

    1. hello_html_3847350f.gif, hello_html_m7eae2593.gif,

    2. hello_html_m6f1f3e2e.gifили hello_html_m75be26e2.gif,

    3. hello_html_m1ad751e1.gif- критические точки функции

    1. 3

    1. Выбираем критические точки, лежащие внутри hello_html_m41cdf12f.gif

    1. hello_html_14868e98.gif

    1. 4

    1. Находим значения функции в критических точках (внутри данного отрезка) и на концах отрезка

    1. hello_html_m64e7d0f0.gif

    2. hello_html_2d1df33.gif

    3. hello_html_619b325c.gif

    1. 5

    1. Из найденных значений функции выбираем наименьшее и наибольшее

    1. hello_html_a4d9f9c.gif, hello_html_4389137e.gif

  3. Примеры. Применяя указанный выше план, найдите наименьшее и наибольшее значения функции hello_html_d00fecf.gif на промежутке hello_html_m41cdf12f.gif, если:

  4. 1) hello_html_131fa472.gif, hello_html_68abf44b.gif; 2) hello_html_m6757c0e6.gif, hello_html_m7d4d8866.gif; 3) hello_html_m1e3192f4.gif, hello_html_771049ff.gif; 4) hello_html_m128eb5b2.gif, hello_html_m432553e9.gif; 5) hello_html_5d89c152.gif, hello_html_22e1f9ac.gif;

  5. 6) hello_html_m6745dda0.gif, hello_html_1ac29183.gif; 7) hello_html_82dde3.gif, hello_html_2d00767a.gif; 8) hello_html_14be1805.gif, hello_html_m5cf09daf.gif;

  6. 9) hello_html_6e1c08a2.gif, hello_html_m39313834.gif.

  1. Геометрические задачи на нахождение оптимальных значений величин.

  1. Задание. Из кружка жести радиуса R вырезается сектор и из оставшейся части круга делается коническая воронка. При какой величине угла вырезаемого сектора объём воронки будет наибольшим?

  2. шага

    1. План решения

    1. Применение плана

    1. 1

    1. Строим рабочий чертеж

    1. 2

    1. Записываем исходную формулу для вычисления величины, экстремальное значение которой требуется найти

    1. hello_html_414b37a7.gif

    1. 3

    1. Вводим переменную величину х и выражаем через неё значения всех величин исходной формулы

    1. Пусть х – величина центрального угла оставшегося сектора, тогда hello_html_m1cb78ab5.gif и hello_html_4830669a.gif, значит hello_html_1a5a06d8.gif и hello_html_m143dcc0d.gif. Высота воронки hello_html_m5531c863.gif

    1. 4

    1. Подставляя найденные значения величин в формулу, представляем её как функцию аргумента х

    1. hello_html_7cc12725.gif,

    2. hello_html_2148ccaf.gif

    1. 5

    1. Задаем (по смыслу задачи) область определения функции

    1. hello_html_2f56209a.gif, hello_html_5259e45c.gif

    1. 6

    1. Функцию аргумента х исследуем на экстремум на найденном числовом промежутке

    1. hello_html_mfc6160f.gif, hello_html_6948f924.gif,

    2. hello_html_m702f0e2b.gif, hello_html_m58c11d8e.gif, hello_html_ab76ba.gif

    1. 7

    1. Записываем ответ

    1. Величина вырезаемого угла равна

    2. hello_html_740c3d9c.gif

  3. ВАРИАНТЫ ПРАКТИЧЕСКОЙ РАБОТЫ.

  4. Вариант 1.

  1. Найдите наибольшее и наименьшее значения функции hello_html_m3357296a.gif на отрезке hello_html_m69427a6c.gif.

  2. Из квадратного листа жести со стороной 12 м надо изготовить бак с квадратным основанием без крышки наибольшего объема. Найдите размеры бака и его объем.

  1. Вариант 2.

  1. Найдите наибольшее и наименьшее значения функции hello_html_352eaea1.gif на отрезке hello_html_2b7efd2c.gif.

  2. Какой из прямоугольников с периметром 2p имеет наибольшую площадь?

  1. Вариант 3.

  1. Найдите наибольшее и наименьшее значения функции hello_html_ae73d21.gif на отрезке hello_html_m3379fc61.gif.

  2. Разность двух чисел равна 8. Каковы должны быть эти числа. Чтобы произведение куба первого числа на второе было наименьшим?

  1. Вариант 4.

  1. Найдите наибольшее и наименьшее значения функции hello_html_352eaea1.gif на отрезке hello_html_7e31e659.gif.

  2. Для стоянки машин выделили площадку прямоугольной формы, примыкающую одной стороной к стене здания. Площадку обнесли с трех сторон металлической сеткой длиной 200 м. И площадь ее при этом оказалась наибольшей. Каковы размеры площадки?

  1. Вариант 5.

  1. Найдите наибольшее и наименьшее значения функции hello_html_ae73d21.gif на отрезке hello_html_5233002a.gif.

  2. Из куска картона 32 см hello_html_m681e791d.gif20 см требуется изготовить открытую сверху коробку наибольшей вместимости, вырезая по углам квадраты и затем, загибая выступы для образования боковых сторон коробки. Найдите объем коробки.

  1. Вариант 6.

  1. Найдите наибольшее и наименьшее значения функции hello_html_m3357296a.gif на отрезке hello_html_m132ce589.gif.

  2. Требуется сделать коробку, объем которой должен равняться 108 смhello_html_m4eb13cb.gif. Коробка открыта сверху и имеет квадратное дно. Каковы должны быть ее размеры, чтобы на ее изготовление пошло наименьшее количество материала?

  1. Вариант 7.

  1. Найдите наибольшее и наименьшее значения функции hello_html_352eaea1.gif на отрезке hello_html_m43b1863b.gif.

  2. На странице книги печатный текст должен занимать (вместе с промежутками между строк)

  1. 160 смhello_html_m7fef8588.gif. Ширина полей на странице слева и справа должна быть равна 2 см, а сверху и снизу – 5 см. Если принимать во внимание только экономию бумаги, то каковы должны быть наиболее выгодные размеры страницы?

  2. Вариант 8.

  1. Найдите наибольшее и наименьшее значения функции hello_html_ae73d21.gif на отрезке hello_html_m161c04a2.gif.

  2. Материальная точка совершает прямолинейное движение по закону hello_html_m48fe4309.gif, где t – время в секундах, s – путь в метрах. В какой момент времени t скорость движения точки будет наибольшей и какова величина этой наибольшей скорости?

  1. Практическая работа № 14.

  2. Применение производной к исследованию функций и построению графиков.

  3. Цель: научиться применять производную при исследовании функций.

  4. Учебные задачи:

  1. Научиться применять производную для исследований функций на монотонность и экстремумы.

  2. Научиться строить графики функций.

  3. Учить рассуждать и логически мыслить.

  1. Краткие теоретические и учебно-методические материалы по теме практического занятия

  2. Исследование функций с помощью производной

  3. Способность производной характеризовать скорость изменения функции (а значит, и ее графика) лежит в основе исследования функций с помощью производной и построения графика.

  4. Для возрастающей функции (рис. 3) угол наклона касательной острый, т.е. f `(x) = tgα > 0, для убывающей (рис. 4) − тупой, т.е. f '(x) = tgα < 0.

  5. Можно по известному знаку производной судить о поведении функции.

  6. hello_html_79b5e18f.jpg

  7. Функция постоянна в каждом интервале, в котором ее производная равна нулю, возрастает в каждом интервале, где производная положительна, и убывает в тех интервалах, где производная отрицательна.

  8. Особую роль играют так называемые критические точки из области определения функции, т.е. точки, в которых производная обращается в нуль, либо не существует.

  9. Среди критических точек отметим точки экстремума.

  10. Экстремумы функции

  11. Говорят, что функция y = f (x) имеет максимум в точке x1 (рис.5), если значение функции в этой точке больше, чем ее значения во всех точках, достаточно близких к x1 , т.е. если f(+∆x) < f(x1) для любых x, как положительных, так и отрицательных, но достаточно малых по модулю. Таким образом, x= x1 точка максимума, a ymax = f (x1) − максимум функции.

  12. Говорят, что функция у = f(x) имеет минимум в точке x2 (рис.5), если значение функции в этой точке меньше, чем ее значения во всех точках, достаточно близких к x2 , т.е. если f(x2 + x)>(х2) для любых x, как положительных, так и отрицательных, но достаточно малых по модулю. Таким образом, x= x2 − точка минимума, a ymin = f (x2) − минимум функции.

  13. hello_html_m48a2f018.jpg

  14. Для исследования функции по первой производной следует:

  15. 1 .Найти область определения функции,

  16. 2.Найти первую производную и критические точки.

  17. 3. Отметить границы области определения и критические точки на числовой прямой.

  18. 4. Исследовать знак производной в каждом из полученных интервалов.

  19. 5. Выписать точки экстремума и вычислить экстремумы функции.

  20. Пример 9. Найти экстремумы функции у = (1 − х )

  21. Решение

  22. 1 .Областью определения функции служит множество всех действительных чисел, т.е.

  23. 2. Функция имеет производную всюду, поэтому определяем критические точки из условия f '(x) = 0. Находим производную.

  24. у' = 3(1 x2 )2 (1 − х2)' = 3(1 − х2)2(−2 х) = −6х(1 − х2)2

  25. у' = 0; −6х(1 − х2)2 = 0 ; x1 = 0; х2= −1; x3=1.

  26. 3. Отметим эти критические точки на числовой прямой (рис.6).hello_html_c2515eb.jpg

  27. 4. Исследуем знак производной у' = −6х(1 − х2)2в каждом из полученных интервалов: у'(−2)> 0, у'(−0,5) > 0, у'(0,5) < 0, у'(−2) < 0.

  28. 5. Точка х = 0 − точка максимума, так как при переходе через нее слева направо производная меняет знак с плюса на минус: ymax = y(0)=1.
    Точки
    х =1 и х = 1 не являются точками экстремума.

  29. Можно провести исследование функции с помощью второй производной.

  30. Если в точке х = х0 первая производная равна нулю (f'(x0) =0), а вторая

  31. производная отлична от нуля, то х = х0точка экстремума.

  32. При этом если вторая производная в этой точке положительна

  33. (f ''(x0) > 0), то х = х0 точка минимума; если вторая производная в этой точке отрицательна (f ''(x0) < 0), то х = х0 точка максимума.

  34. Для исследования функции на экстремум по первой и второй производной следует:

  1. Найти область определения функции.

  2. Найти первую производную функции и стационарные точки, т.е. точки, в которых она обращается в нуль.

  3. Найти вторую производную и исследовать ее знак в каждой критической точке.

  4. Выписать точки экстремума и вычислить (если нужно) экстремумы функции.

  1. hello_html_177dff42.jpg

  2. Пример 10. Найти экстремумы функции

  3. Решение

  4. 1. Областью определения функции служит множество всех действительных чисел, т.е.

  5. 2. Функция имеет производную всюду, поэтому критические точки определяем из условия f ' (x) = 0.

  6. hello_html_17821ac9.jpg

  7. 3. Находим вторую производную функции f '' (x) = 6 x − 6 . Исследуем знак второй производной в каждой критической точке: f '' (0) = − 6 < 0 , значит, x=0 - точка максимума, ymax = y (0) = 1.
    f '' (2) = 6 > 0, значит, x=2 - точка минимума ymin = y (2)= 23 − 3·22 + 1 = −3.

  8. Пример. Исследовать функцию и построить ее график:

  9. Решение:

  1. .

  2. Функция нечетная, т.к. . Следовательно, она симметрична относительно начала координат.

  3. Точки пересечения графика с осью ОХ:;

  1. Точки пересечения графика с осью ОY:.

  1. Исследуем функцию на монотонность и точки экстремума:

  1. ,

  2. Функция возрастает на; функция убывает на .

  3. - точка максимума, - точка минимума.

Исследуем функцию на вогнутость, выпуклость и точки перегиба:
  1. Функция вогнута на, выпукла на .

  2. - точка перегиба.

Построим график функции:
  1. hello_html_m39422f81.jpg.

    1. а)

    1. а)

    1. а)

    1. б)

    1. б)

    1. б)

    1. в)

    1. в)

    1. в)

    1. г)

    1. г)

    1. г)

    1. Исследовать функции и построить их графики

    1. а)

    1. а)

    1. а)

    1. б)

    1. б)

    1. б)

    1. в)

    1. в)

    1. в)

  2. Общая схема исследования функций с помощью производной

  1. Нахождение области определения функции.

  2. Проверка того, является ли функция четной, нечетной, периодической или эта функция – функция общего вида.

  3. Определение точек пересечения с осями координат.

  4. Нахождение критических точек (точек, в которых производная равна нулю или не существует).

  1. Определение промежутков возрастания и убывания функции

  1. (промежутков, на которых производная положительна или отрицательна).

  1. Определение экстремумов функции.

  2. Исследование функции на выпуклость, вогнутость, определение точек перегиба (исследование проводится по второй производной функции).

  1. 9. Уточнение графика функции по точкам (произвести окончательное уточнение графика, в особенности на участках, где информация о нем недостаточна).

  2. Вопросы для закрепления теоретического материала к практическому занятию:

  1. Что такое интервалы монотонности?

  2. Что такое max и min для функции?

  3. Вспомнить алгоритм исследования функции на экстремумы.

  1. Порядок выполнения отчёта по практической работе:

  2. 1. Выполнить задания.

  3. 2. Ответить на вопросы для закрепления теоретического материала (устно).

  4. 3. Оформить отчёт по практической работе.

  5. Образец отчёта по практической работе:

  6. Тема.

  7. Учебная цель.

  8. Название практической работы.

  9. Решение заданий практической работы.

  10. Ответы на вопросы для закрепления теоретического материала.

  11. Практическая работа № 15 -16.

  12. Нахождение неопределенного интеграла.

  13. Цель: Сформировать навыки простейшего интегрирования с помощью таблицы интегралов.

  14. Учебные задачи:

  1. Закрепить умение находить неопределенные интегралы для элементарных функций.

  2. Учить рассуждать и логически мыслить.

  1. Краткие теоретические и учебно-методические материалы по теме практического занятия

  2. Таблица интегралов

    1.
  3. 2.

  4. 3.

  5. 4.

  6. 5.

  7. 6.

    1. 7.

    2. 8.

    3. 9.

    4. 10.

    5. 11.

    6. 12.

    1. 13.

    2. 14.

    3. 15.

    4. 16.

  8. Действие вычисления неопределенного интеграла данной функции называется интегрированием данной функции.

  9. Очевидно, что действие интегрирования обратно по отношению дифференцированию.

  10. Вопросы для закрепления теоретического материала к практическому занятию:

  1. Что такое неопределенный интеграл?

  2. Как проверить результаты интегрирования?

  3. Сколько первообразных может быть у одной функции? Как называется это свойство?

  1. Задания для практического занятия:

    1) ;
  2. 2) ;

  3. 3) ;

  4. 4) ;

  5. 5) ;

  6. 6) ;

    1. 1) ;

    2. 2) ;

    3. 3) ;

    4. 4) ;

    5. 5) ;

    6. 6) .

    1. 2.Вычислите интеграл

    1. 1) ;

    2. 2) ;

    3. 3) ;

    4. 4) ;

    1. 1) ;

    2. 2) ;

    3. 3) ;

    4. 4) .

  7. Инструкция по выполнению практического занятия

  8. Для решения практических заданий рассмотрите пример.

  9. Пример 1. Пользуясь таблицей основных интегралов и свойствами неопределенного интеграла, найти интегралы.

  10. Решение.

  11. Порядок выполнения отчёта по практической работе:

  12. 1. Выполнить задания.

  13. 2. Ответить на вопросы для закрепления теоретического материала (устно).

  14. 3. Оформить отчёт по практической работе.

  15. Образец отчёта по практической работе:

  16. Тема.

  17. Учебная цель.

  18. Название практической работы.

  19. Решение заданий практической работы.

  20. Ответы на вопросы для закрепления теоретического материала.

  21. Практическая работа № 17.

  22. Нахождение площади криволинейной трапеции.

  23. Цель работы: закрепить навыки применения определенного интеграла к решению прикладных задач.

  24. Выполняя данную работу, студент должен

  25. знать:

  • формулы табличных интегралов;

  • формулу Ньютона – Лейбница

  • формулу пути, пройденного точкой;

  • формулу площади плоской фигуры.

  1. уметь:

  • с помощью формул, перечисленных выше, решать задачи прикладного характера.

  1. Последовательность выполнения:

  2. задания выполнять желательно в указанном порядке.

  3. Методические указания:

    При решении задачи применить формулу пути, пройденного точкой: . Применить формулу площади плоской фигуры: .
  4. Теоретический материал

  5. Таблица первообразных:

    Критерии оценивания:
  6. На оценку «5» - необходимо выполнить все задания работы с грамотным оформлением условий, чертежей и решений задач;

  7. На оценку «4» - выполнить задания №1, №2; №3(а,в);

  8. На оценку «3» - выполнить задания №1; №2; №3(а,б).

  9. Задания

    1. 1 вариант

    2. 1. Определить путь S, пройденный точкой за время t = 3 с от начала движения, если скорость материальной точки, движущейся прямолинейно, равна .

    3. 2. Вычислить площадь фигуры, изображенной на рисункеhello_html_mbd46a71.png

    4. 3. Вычислить площадь фигуры , ограниченной заданными линиями и сделать чертеж:

    5. а) у = 4-х2, у = 0, x = 1, x = 3;

    6. б) ;

    7. в) .

    8. 2 вариант

    9. 1. Определить путь S, пройденный точкой за время t = 2 с от начала движения, если скорость материальной точки, движущейся прямолинейно, равна .

    10. 2.Вычислить площадь фигуры, изображенной на рисункеhello_html_m767af3f.png

    11. 3.Вычислить площадь фигуры, ограниченной заданными линиями и сделать чертеж:

    12. а) у = 4х-х2, у = 0, x = 1, x = 3;

    13. б)

    14. в)

    15. 3 вариант

    16. 1. Определить путь S, пройденный точкой за время t = 1 с от начала движения, если скорость материальной точки, движущейся прямолинейно, равна .

    17. 2.Вычислить площадь фигуры, изображенной на рисункеhello_html_m71e1446b.png

    18. 3. Вычислить площадь фигуры, ограниченной заданными линиями и сделать чертеж:

    19. а) у = 3х2, у = 0, х = 0, х = 2.

    20. б)

    21. в)

    22. 4 вариант

    23. 1. Определить путь S, пройденный точкой за время t = 4 с от начала движения, если скорость материальной точки, движущейся прямолинейно, равна .

    24. 2.Вычислить площадь фигуры, изображенной на рисункеhello_html_35f45859.png

    25. 3.Вычислить площадь фигуры, ограниченной заданными линиями и сделать чертеж:

    26. а) у = х2+2х+1, у = 0, х=3, х=1.

    27. б)

    28. в)

    1. Практическая работа № 18.

    2. Вычисление интегралов. Нахождение площади криволинейной трапеции.

    3. ЦЕЛЬ РАБОТЫ:

    1. Корректировать знания, умения и навыки в теме: «Применение определенного интеграла для вычисления площадей и объемов».

    1. Закрепить и систематизировать знания по теме.

    1. Определить уровень усвоения знаний, оценить результат деятельности уч-ся.

    1. ОБОРУДОВАНИЕ: инструкционно-технологические карты, таблицы первообразных некоторых функций, микрокалькуляторы.

    2. ПОРЯДОК ВЫПОЛНЕНИЯ РАБОТЫ:

    1. Ответить на контрольные вопросы:

    1. а) Какую фигуру называют криволинейной трапецией?

    2. Приведите примеры криволинейных трапеций.

    3. б) Запишите формулу для вычисления площади криволинейной трапеции.

    4. в) Покажите на рисунках и запишите интегральные формулы, с помощью которых можно вычислить площади фигур, не являющихся криволинейными трапециями.

    5. г) Запишите и с помощью иллюстрации прокомментируйте интегральную формулу для вычисления объемов тел.

    1. С помощью обучающей таблицы повторить план вычисления площади криволинейной трапеции и изучить образцы решенных задач.

    1. Выполнить задания для самоконтроля (в таблице).

    2. Изучить условие заданий для практической работы.

    1. Оформить отчет о работе.

    1. ОБУЧАЮЩАЯ ТАБЛИЦА

    2. Определение. Криволинейной трапецией называется фигура, ограниченная графиком непрерывной и не меняющей на отрезке a;b знака функции f ( x ) , прямыми x a, x b и отрезком a;b. Площадь S криволинейной трапеции находится по формуле

    3. b

    4. S f ( x )dx F( b ) F( a ) . (*)

    5. a

    6. Задание. Вычислите площадь фигуры, ограниченной линиями:

    7. а) y hello_html_27f35a92.jpghello_html_m62154990.jpg x , y 2, x 9 ; б) y x 2 , y 2 x, y 0 .

    8. hello_html_6d4113bc.jpg

    9. y
      1. , y 2, x 9

      1. б)

      1. x

      1. а

      1. y x

      1. 2

      1. , y 2

      1. x, y 0

    1. Строим заданные линии и

    1. штриховкой отмечаем фигуру, площадь которой надо найти. Установим, является ли эта фигура криволинейной трапецией

    2. 2
      1. Записываем

      1. формулу для

      1. S S ABCDE S ABDE

      1. S SOAC S ACB

      1. вычисления

      1. площади

      1. искомой фигуры

      1. b

      1. b

      1. a

      1. b

      1. xdx 2dx

      1. x 2 dx ( 2 x )dx

      1. a

      1. a

      1. 0

      1. a

      1. 3

      1. Находим

      1. пределы

      1. 2

      1. y

      1. x ,

      1. x 2

      1. x 4

      1. ,

      1. y x

      1. ,

      1. интегрирования

      1. y 2;

      1. y 2 x;

      1. a x A 4, b xB 9

      1. x 2 x 2 0 x 2;1

      1. 4

      1. Вычисляем

      1. искомую

      1. 9

      1. 9

      1. 3

      1. 9

      1. 1

      1. 2

      1. 2x

      1. 2

      1. S x 2 dx ( 2 x )dx

      1. площадь по формуле (*)

      1. S

      1. xdx 2dx

      1. 3

      1. 4

      1. 4

      1. 0

      1. 1

      1. 4

      1. 9

      1. 2

      1. x3

      1. 1

      1. x 2

      1. 2

      1. 1

      1. 2x

      1. ( 27

      1. 8 ) 2( 9

      1. 4 )

      1. 2x

      1. 4

      1. 3

      1. 3

      1. 2

      1. 3

      1. 0

      1. 1

      1. 8

      1. ,

      1. 3

      1. 4

      1. 1

      1. 5

      1. ,

      1. 4

      1. 2

      1. 2

      1. 6

      1. S 2

      1. 2

      1. (кв.ед.)

      1. 2

      1. 3

      1. S

      1. 5

      1. (кв.ед.)

      1. 6

    3. hello_html_m4c65793d.jpghello_html_m5168c726.jpg

    4. Примеры. Вычислите площадь фигуры, ограниченной линиями:

    5. , y 0, x 2 ; 2) y x 2 , y 1; 3) y x 2 1, y 0 ;4) y 1 x 2 , y 2 ;
      1. 5)

      1. y e x , y 0, x 0, x 1; 6) y x 3 , y

      1. ; 7) y 2x x 2 , y

      1. 3

      1. ; 8)

      1. x

      1. 4

      1. y x3 , y 1, x 2 ;

      1. 9)

      1. y

      1. 5

      1. , y 6 x .

      1. x

    6. hello_html_m4c65793d.jpg

    7. ВАРИАНТЫ ПРАКТИЧЕСКОЙ РАБОТЫ.

    8. Вариант 1.

    1. Вычислите площадь фигуры, ограниченной линиями:

    1. y x 2 3x 4, y x 1 .

    1. Выберите правильный вариант ответа.

    1. Площадь фигуры, изображенной на

    2. hello_html_4078e979.jpg

    3. рисунке, вычисляется по формуле:

    4. а) S 2x 2 2dx 2 ;

    5. 2

    6. б) S 1x 2 2dx 2 ;

    7. 1

    8. в) S 1x 2 2dx 2 .

    9. 2

    10. Вариант 2.

    1. Вычислите площадь фигуры, ограниченной линиями:

    1. y 0,5x 2 2x 3, y 7 x .

    1. Выберите правильный вариант ответа Площадь фигуры, изображенной на

    1. hello_html_661e7c7e.jpg

    2. рисунке, вычисляется по формуле:

    3. а) S 1x 2 4dx 3 ;

    4. 2

    5. б) S 2 x 2 4dx 3 ;

    6. 2

    7. в) S 14 x 2 dx 3 .

    8. 2

    9. Вариант 3.

    10. 1. Вычислите площадь фигуры, ограниченной линиями: y x 22 , y 4 x 2 .

    1. Выберите правильный вариант ответа. Площадь фигуры, ограниченной линиями y hello_html_27f35a92.jpghello_html_m62154990.jpg x 1, y 2, y 0, x 0 , равна:

      1. 2

      1. ; б) 4; в) 3

      1. 1

      1. .

      1. 3

      1. 3

      1. Вариант 4.

      1. 1. Вычислите

      1. площадь

      1. фигуры,

      1. ограниченной

      1. линиями:

    1. y x 2 2x 2, y 2 4x x 2 .

    1. Выберите правильный вариант ответа. Площадь фигуры, ограниченной линиями y hello_html_m11c68f66.jpghello_html_4072026d.jpg x 1, y 2, y 0, x 2 , равна:

      1. 1

      1. ;

      1. б) 3

      1. 2

      1. ;

      1. в) 4

      1. 2

      1. .

      1. 3

      1. 3

      1. 3

    1. Вариант 5.

    1. Вычислите площадь фигуры, ограниченной линиями:

    1. y x 2 2x 4, y 3, x 1.

    1. Выберите правильный вариант ответа. Площадь фигуры, ограниченной

    1. линиями y e 2x , y 0, x 0, x a a 0, равна e 1 , если, а равно:

    2. .
      1. 2

      1. 2e

    3. Вариант 6.

    4. 1. Вычислите площадь фигуры, ограниченной линиями: y 4x x 2 , y 4 x .

    1. Выберите правильный вариант ответа. Площадь фигуры, ограниченной линиями y 2e0,5x , y 0, x 0, x b b 0, равна 4e 2 4 , если b равно:

    1. а)

    2. 2e

    3. ;

    4. б) 4;

    5. в)

    6. 4 .e

    1. Практическая работа № 19.

    2. Решение уравнений и неравенств.

    3. ЦЕЛЬ РАБОТЫ: систематизировать знания обуч-ся в теме: «Решение рациональных уравнений и неравенств», организовать деятельность по переводу своих знаний от усвоения отдельных фактов и понятий к их обобщению в целостную систему знаний, определить уровень усвоения знаний, оценить результат деятельности обуч-ся.

    4. ПОРЯДОК ВЫПОЛНЕНИЯ РАБОТЫ:

    1. Выполнить обучающий тест и проверить свои результаты по таблице ответов.

    1. Изучить условие заданий для практической работы.

    2. Оформить отчет о работе.

    1. УКАЗАНИЯ К ВЫПОЛНЕНИЮ ПРАКТИЧЕСКОЙ РАБОТЫ

    2. пример 1. Решите неравенство x 32 x 73 x 1 0 .

    3. x 2x 44

    4. РЕШЕНИЕ. Это рациональное неравенство решим методом интервалов. Отметим на числовой прямой «жирными» точками нули числителя (–1; 3 и 7)

    • «прозрачными» – нули знаменателя (–4 и 2). Если бы заданное неравенство было строгим, нужно было бы все нули сделать «прозрачными». Эти точки разобьют числовую прямую на 6 интервалов:

    1. hello_html_m187ac648.jpg

    2. Выясним знак данной дроби на каждом из этих интервалов, используя пробные числа, принадлежащие интервалам.

    3. Можно поступать иначе. Для этого в выражении в каждом из множителей переменная х должна иметь знак «+» ((х – 2), а не (2 – х); (х – 7),

    • не (7 – х)). Этого всегда можно добиться, умножая неравенство на –1 и меняя одновременно его знак столько раз, сколько надо. Отметив нули выражения на числовой оси, справа налево расставим знаки по следующему правилу: сначала «+», меняем знак на нечетной степени и сохраняем его на четной.

    1. Теперь остается выписать ответ – промежутки, на которых поставлен знак «+», так как знак данного неравенства . Важно не забыть х = 3.

    2. ОТВЕТ: 1; 2 37;.

    3. ПРИМЕР 2. Решите неравенство x 2 x 1 0 .

    4. РЕШЕНИЕ. Это квадратное неравенство можно решить методом интервалов, но проще – графически. Рассмотрим функцию, заданную уравнением y x 2 x 1. Графиком ее является парабола. Заметим, что для нас совершенно не важны точные характеристики параболы (где находится ось, пересечение с Оу и т. п.) Достаточно знать, что ее ветви направлены вверх (а

    • 0) и что она пересекает ось Ох в двух точках, являющихся корнями

    1. 1
      1. 5

      1. , x

      1. 1

      1. 5

      1. . Выполним схематический

      1. 1

      1. 2

      1. 2

      1. 2

    2. hello_html_m6c21420e.jpghello_html_be7693d.jpg

    3. рисунок:

    4. hello_html_m220cbf91.jpg

    5. Из рисунка видно, что квадратичная функция принимает положительные значения вне отрезка, соединяющего ее корни.

    1. ОТВЕТ: ;

    2. 1
      1. 1

      1. 5

      1. 5

      1. ;

      1. .

      1. 2

    3. hello_html_39d210c4.jpghello_html_39d210c4.jpg

    1. ЗАДАНИЯ ДЛЯ ПРАКТИЧЕСКОЙ РАБОТЫ

    1. Вариант 1

    1. Решите неравенства:

    1. а) 4 x2 x 62 x 52 2 x2 ;

    2. x 2
      1. 2x

      1. 3x 10

      1. ;

      1. 3

      1. 4

      1. 3

    3. в) x 22 x 3x 6 0 .

    1. Решите систему неравенств:

    1. 5x 1 9x 3 6x 2, 33 2x 7x 2x 8.

    2. Вариант 3

    1. Решите неравенства:

    1. а) 3 2x 8 5x 2 x ;

    2. 5 2

    3. б) xx 1 21 2x x 2 ;

    4. в) x 52 2 x3 0 .

    5. 2. Решите систему неравенств:

    6. 2x 1 3x 2 x,

    7. Вариант 5

    1. Решите неравенства:

    1. а) x 32 xx 2 3 ;

    2. Вариант 2

    1. Решите неравенства:

    1. а) 5x 1 x7 x x 2 ;

    2. б) x 2 2 7x ;

    3. 10 10

    4. в) x 82 10 x3 0 .

    1. Решите систему неравенств:

    x 9
    1. x 1

    1. x 2

    1. x

    1. ,

    1. 4

    1. 2

    1. 3

    1. 2 x 2x 8.

    1. Вариант 4

    1. Решите неравенства:

    1. а) 5 x 7 4x 1 ;

    2. 3 2 8

    3. б) 2xx 1 3x 1;

    4. в) x 53 x 44 0 .

    5. 7 x5

    6. 2. Решите систему неравенств:

    7. 3,3 31,2 5x 0,610x 1, 1,6 4,54x 1 2x 26,1.

    8. Вариант 6

    1. Решите неравенства:

    1. а) x 12 x 42 6 x2 3 x2 ;

    2. б) 4x 2x 2 5 0 ;

    4x 5 0 ;
    1. в)

    1. x 2

    1. 4x 5

    1. 0 .

    1. x 2

    1. 2x 3

    1. x 2

    1. 3x 2

    1. в)

    1. 0 .

    1. x 7

    1. 2. Решите систему неравенств:

    1. 2

    1. 101

    1. 6x,

    1. 2. Решите систему неравенств:

    1. xx 1 x

    1. 3,5 x 1,5 6 4x.

    1. 5,81 x1,86 x 5,

    2. Практическая работа № 20-21.

    3. Решение задач на перебор вариантов, на подсчет числа размещений, перестановок, сочетаний.

    4. Цель работы: закрепить знания и навыки в решении комбинаторных задач

    5. Умение и навыки, которые должны приобрести студенты на занятии: вычислять сочетания, размещения, перестановки.

    6. Ход работы.

    7. 1. Самостоятельно рассмотреть решение типовых примеров, используя основные теоретические сведения.

    8. 2.Выполнить расчеты по дидактическим карточкам-заданиям.

    9. 3.Составить отчет: оформить решение практических задач в тетради.

    10. Приложение: дидактические карточки с заданиями 4 вариантов.

    11. Основные теоретические сведения

    12. Основные понятия комбинаторики

    13. Определение.

    14. Произведение всех натуральных чисел от 1 до n включительно называют

    15. n- факториалом и пишут

    16. hello_html_m4055214c.gif.

    17. Перестановки.

    18. Комбинация из n элементов, которые отличаются друг от друга только порядком элементов, называются перестановками.

    19. Перестановки обозначаются символом Рn, где n- число элементов, входящих в каждую перестановку. (Р - первая буква французского слова permutation- перестановка).

    20. Число перестановок можно вычислить по формуле

    21. hello_html_m3b1da20e.gif

    22. или с помощью факториала:

    23. hello_html_m68104c69.gif

    24. 0!=1 и 1!=1.

    25. Размещения.

    26. Определение. Размещениями из m элементов в n в каждом называются такие соединения, которые отличаются друг от друга либо самими элементами (хотя бы одним), либо порядком из расположения.

    27. Размещения обозначаются символом hello_html_m67e70b0a.gif, где m- число всех имеющихся элементов, n- число элементов в каждой комбинации.

    28. При этом полагают, что nhello_html_m3f2066f7.gifm.

    29. hello_html_6809de7a.gif

    30. Сочетания.

    31. Определение. Сочетаниями называются все возможные комбинации из m элементов по n, которые отличаются друг от друга по крайней мере хотя бы одним элементом (здесь m и n-натуральные числа, причем n hello_html_m3f2066f7.gif m).

    32. hello_html_36f4f892.gif

    33. Число сочетаний из m элементов по n обозначаются hello_html_154aa149.gif .

    34. В общем случае число из m элементов по n равно числу размещений из m элементов по n, деленному на число перестановок из n элементов:

    35. hello_html_m562004e3.gif

    36. Правило суммы.
      Если элемент a можно  выбрать m способами, а элемент b – n способами, причем любой выбор элемента a отличен от любого выбора элемента  b, то выбор  “a или b”  можно

    37. сделать  m + n  способами.
      Правило произведения.
      Если из некоторого множества А элемент  a
      i  можно выбрать КA способами, а элемент bj  из множества  В – КB  способами, то совокупность (ai ; bj ) можно образовать КA* КB  способами. Правило верно и для совокупностей, состоящих из большего, чем два числа элементов.

    38. Перестановки с повторением.
      Иногда требуется переставлять предметы, некоторые из которых неотличимы друг от друга. Рассмотрим такой вариант перестановок, который называется перестановками с повторениями.
      Пусть имеется
      п1 предметов 1-го типа, n2 предмета 2-го, пк предметов hello_html_m256ddab7.png-го типа и при этом п1+ п2+...+ пк = п. Количество разных перестановок предметов
      hello_html_1becaa0b.png                                

    39. Размещения с повторениями.

    40. Пусть даны элементы а, а, . . . , аn     (а)

    41. Размещением с повторениями из n элементов по k элементов называется всякая упорядоченная последовательность из k элементов, членами которой являются данные элементы. В размещении с повторениями один и тот же элемент может находиться на нескольких различных местах.
      Формула для числа размещений с повторениями.
      Каждый элемент может быть выбран  n способами, поэтому :

    42. hello_html_m5ceca8e8.png= hello_html_29b953ab.png,где hello_html_m5ceca8e8.png -обозначение размещений с повторениями .

    43. Примеры типовых расчетов: выполняется всей группой вместе с преподавателем.

    44. Пример 1. Сколькими способами можно расставлять на одной полке шесть различных книг?

    45. Решение. Искомое число способов равно числу перестановок из 6 элементов, т.е.

    46. hello_html_203fdcd0.gif.

    47. Пример 2. Сколько вариантов распределения на практику в три ресторана различного профиля можно составить для пяти студентов?

    48. Решение. Искомое число вариантов равно числу размещений из 5 элементов по 3 элемента, т.е.

    49. hello_html_7b070a4c.gif.

    50. Пример 3. Из группы в 25 человек нужно выделить четырех для работы официантами на банкете. Сколькими способами это можно сделать?

    51. Решение. Так как порядок выбранных четырех человек не имеет значения, то это можно сделать hello_html_m598bbfab.gif способами.

    52. Находим по первой формуле

    53. hello_html_7d2711c8.gif.

    54. Приложение. Дидактические карточки-задания к практической работе.

    55. Вариант 1

    56. 1.Вычислить hello_html_617034be.gifhello_html_m33b2e3f.gif

    57. 2.Упростить hello_html_m500fb29d.gif

    58. 3.Вычислить hello_html_8d79d56.gif

    59. 4.Вычислить hello_html_m1026c7c6.gif; hello_html_m401b0fb6.gif

    60. 5.Сколькими способами могут разместиться 5 человек вокруг круглого стола?

    61. 6.Сколько двузначных чисел можно составить из цифр 1,2,3,8,9 так, чтобы в каждом числе не было одинаковых цифр?

    62. 7.Решить уравнение hello_html_716aa33b.gif

    63. Вариант 2

    64. 1.Вычислить hello_html_2e99e841.gif

    65. 2.Упростить hello_html_m6c976d32.gif

    66. 3.Вычислить hello_html_m36d3d0ab.gif

    67. 4.Вычислить hello_html_1af11a.gif; hello_html_m14ec6358.gif

    68. 5.Сколькими способами можно расставить на полке 6 книг?

    69. 6.Сколько флажков 3 разных цветов можно составить из 5 флажков разного цвета?

    70. 7.Решить уравнение hello_html_m3dc3dbb8.gif

    71. Вариант 3

    72. 1.Вычислить hello_html_m5b0b0863.gif

    73. 2.Упростить hello_html_42171638.gif

    74. 3.Вычислить hello_html_1532575.gif

    75. 4.Вычислить hello_html_m7fc6baf2.gif; hello_html_5b7b8dec.gif

    76. 5. Сколькими способами можно выбрать 3х дежурных, если в классе 30 человек?

    77. 6.Решить уравнение hello_html_m11546b91.gif

    78. Вариант 4

    79. 1.Вычислить hello_html_46c92293.gif

    80. 2.Упростить hello_html_3ca6b7c4.gif

    81. 3.Вычислить hello_html_m42aa996.gif

    82. 4.Вычислить hello_html_1af11a.gif; hello_html_m7286d096.gif

    83. 5. Сколько вариантов распределения 3х путевок в санаторий различного профиля можно составить для 5 претендентов?

    84. 6.Решить уравнение hello_html_m5f072850.gif

    85. Оформить отчет.

    86. Практическая работа № 22.

    87. Дискретная случайная величина, закон ее распределения.

    88. Цель работы: Нахождение закона распределения, функции распределения и числовых ха-рактеристик случайной величины.

    89. Содержание работы:

    90. Основные понятия.

    1. Случайной называют величину, которая в результате испытания примет одно и только одно возможное значение, наперед не известное и зависящее от случайных причин, которые зара-

    1. нее не могут быть учтены.

    1. Дискретной случайной величиной (ДСВ) называют такую величину, множество значений которой либо конечное, либо бесконечное, но счетное.

    1. Непрерывной случайной величиной (НСВ) называют случайную величину, которая мо-

    1. жет принимать все значения из некоторого конечного или бесконечного промежутка. Множество возможных значений непрерывной случайной величины бесконечно и несчетно.

    1. Заданное соответствие между возможными значениями СВ и их вероятностями называ-

    1. ется законом распределения случайной величины ; его можно задать таблично, аналитически (в виде формулы) и графически.

    1. При табличном задании закона распределения дискретной случайной величины первая строка таблицы содержит возможные значения, а вторая - их вероятности. Эта таблица называется рядом распределения.

    1. Ряд распределения можно представить графически, если по оси абсцисс отложить воз-

    1. можные значения ДСВ, а по оси ординат - соответствующие вероятности. Соединив полученные точки отрезками, получим ломаную, называемую многоугольником распределения вероятностей

    1. Функцией распределения случайной величины Х (обозначается F(x)) называется функ-

    1. ция, определяемая соотношением F(x) = P(X < x).

    1. Математическое ожидание ДСВ Х равно сумме произведений всех ее возможных значе-

    1. n

    2. ний на их вероятности, т.е. М ( Х ) xi Pxi

    1. 1

    1. Дисперсией ДСВХ (D(X)) называют математическое ожидание квадрата отклонения СВ

    1. X M X 2
      1. n

      1. xi M ( X )2 Pxi

      1. i 1

    1. Средним квадратическим отклонением случайной величины Х называется арифметиче-

    1. ский корень из дисперсии, т.е. ( X ) hello_html_78dfff34.jpghello_html_m11f7afd5.jpgD( Х )

    2. hello_html_m6ffc3fd2.jpg

    3. Пример выполнения:

    4. исходные данные:

    5. Приживаемость саженцев яблонь составляет 80%. Наудачу выбирают 5 саженцев. Соста-вить закон распределения числа прижившихся саженцев, функцию распределения, построить мно-гоугольник распределения и график функции распределения. Найти математическое ожидание, дисперсию и среднее квадратическое отклонение числа прижившихся саженцев.

    6. Решение:

    7. Вероятность приживаемости яблони равна 0,8.

    8. Х – случайная величина числа прижившихся яблонь из пяти саженцев:

    9. Возможные значения: х1 = 0 – ни один саженец не прижился;

    10. х2 = 1 один саженец прижился;

    11. х3 = 2 два прижились;

    12. х4 = 3 три;

    13. х5 = 4 четыре;

    14. х6 = 5 пять саженцев прижились.

    15. Вероятности этих значений вычислим по формуле Бернулли:

    16. P(x1) C50 p0q5 5!5!0! 0,80 0,25 0,00032

    17. P(x2 ) C51 p1q4 45!!1! 0,810,24 5 0,8 0,0016 0,0064

    18. P(x3 ) C52 p2q3 25!!3! 0,82 0,23 10 0,64 0,008 0,0512

    19. P(x4 ) C53 p3q2 3!5!2! 0,83 0,22 10 0,512 0,04 0,2048

    20. P(x5 ) C54 p4q1
      1. 5!

      1. 0,84

      1. 0,21 5 0,4096 0,2 0,4096 P(x6 ) C55 p5q0

      1. 5!

      1. 0,85

      1. 0,20 0,32768

      1. 5!0!

      1. 4!1!

      1. Таким образом, закон распределения случайной величины:

      1. Х

      1. 0

      1. 1

      1. 2

      1. 3

      1. 4

      1. 5

      1. Р(Х)

      1. 0,00032

      1. 0,0064

      1. 0,0512

      1. 0,2048

      1. 0,4096

      1. 0,32768

    21. Многоугольник распределения:

    22. hello_html_3572a408.jpg

    23. 0,5
      1. 0,4

      1. 0,3

      1. 0,2

      1. 0,1

      1. 0

      1. 0

      1. 1

      1. 2

      1. 3

      1. 4

      1. 5

    24. Вычислим функцию распределения:

    25. если 0 х 1
      1. 0,00032

      1. 0,0064

      1. 0,00672; если 1 x 2

      1. 0,0064

      1. 0,0512 0,05792; если 2 x 3

      1. F ( X ) 0,00032

      1. 0,00032 0,0064

      1. 0,0512

      1. 0,2048 0,26272; если 3 x 4

      1. 0,0064

      1. 0,0512

      1. 0,2048 0,4096 0,67232; если 4 x 5

      1. 0,00032

      1. 0,0064

      1. 0,0512

      1. 0,2048 0,4096 0,32768 1; если х 5

      1. 0,00032

    26. Найдем числовые характеристики случайной величины, для этого составим таблицу:

    27. 2
      1. 3

      1. 4

      1. 5

      1. Р(Х)

      1. 0,00032

      1. 0,0064

      1. 0,0512

      1. 0,2048

      1. 0,4096

      1. 0,32768

      1. Х-М(Х)

      1. -4

      1. -3

      1. -2

      1. -1

      1. 0

      1. 1

      1. (Х-М(Х))2

      1. 16

      1. 9

      1. 4

      1. 1

      1. 0

      1. 1

      1. 6

      1. Мат. ожидание:

      1. М ( Х ) xi Pxi

      1. i 1

      1. 10,0064 2 0,0512 30,2048 4 0,4096 5 0,32768 4

      1. 6

      1. Дисперсия: D( Х ) xi M ( X )2 Pxi

      1. i 1

      1. 16 0.00032 9 0,0064 4 0,0512 10,2048 0 0,4096 10,32768 0,8

      1. Среднее квадратическое отклонение: ( X )

      1. 0,89.

      1. D( Х )

    28. hello_html_m3a9ac4e.jpg

    29. Задания к практической работе.

    1. В партии из 10 деталей имеется 8 стандартных. Наудачу отобраны 2 детали. Составить закон распределения числа стандартных деталей среди отобранных. Найти функцию распределе-

    1. ния, математическое ожидание, дисперсию и среднее квадратическое отклонение. Построить по-лигон полученного распределения.

    1. Устройство состоит из трех независимо работающих элементов. Вероятность отказа каж-

    1. дого элемента в одном опыте равна 0,1. Составить закон распределения числа отказавших эле-ментов в одном опыте. Построить полигон полученного распределения. Найти функцию распре-деления, математическое ожидание, дисперсию и среднее квадратическое отклонение

    1. Вероятность того, что стрелок попадет в мишень при одном выстреле, равна 0,7. Стрелок делает выстрелы до первого промаха. Составить закон распределения случайной величины Х

    1. числа патронов, выданных стрелку, если всего имеется пять патронов. Построить полигон полу-ченного распределения. Найти функцию распределения, математическое ожидание, дисперсию и среднее квадратическое отклонение

    1. Определить закон, функцию распределения, математическое ожидание, дисперсию и среднее квадратическое отклонение числа гербов при четырех подбрасываниях монеты. Постро-

    1. ить полигон полученного распределения.

    1. Два носка выбираются случайным образом из ящика, в котором находится 5 коричневых

    • 3 зеленых. Определить закон, функцию распределения, математическое ожидание, дисперсию и среднее квадратическое отклонение числа коричневых носков. Построить полигон полученного распределения.

    1. В ящике находится 35 кондиционных и 12 бракованных однотипных деталей. Опреде-

    1. лить закон, функцию распределения, математическое ожидание, дисперсию и среднее квадратиче-ское отклонение количества бракованных деталей среди трѐх наудачу выбранных. Построить по-лигон полученного распределения.

    1. В ящике находится 35 кондиционных и 12 бракованных однотипных деталей. Опреде-

    1. лить закон, функцию распределения, математическое ожидание, дисперсию и среднее квадратиче-ское отклонение количества кондиционных деталей среди трѐх наудачу выбранных. Построить полигон полученного распределения.

    1. В партии из 25 изделий 5 изделий имеют скрытый дефект. Определить закон, функцию распределения, математическое ожидание, дисперсию и среднее квадратическое отклонение коли-

    1. чества дефектных деталей среди трѐх наудачу выбранных. Построить полигон полученного рас-пределения.

    1. В партии из 25 изделий 5 изделий имеют скрытый дефект. Определить закон, функцию распределения, математическое ожидание, дисперсию и среднее квадратическое отклонение коли-

    1. чества качественных деталей среди трѐх наудачу выбранных. Построить полигон полученного распределения.

    1. В городе имеются 4 оптовые базы. Вероятность того, что требуемого сорта товар отсут-

    1. ствует на этих базах, одинакова и равна 0,3. Составить закон распределения, функцию распреде-ления, математическое ожидание, дисперсию и среднее квадратическое отклонение числа баз, на которых искомый товар отсутствует в данный момент. Построить полигон полученного распреде-ления.

    2. 11 В городе имеются 4 оптовые базы. Вероятность того, что требуемого сорта товар отсутствует на этих базах, одинакова и равна 0,3. Составить закон распределения, функцию распределения, математическое ожидание, дисперсию и среднее квадратическое отклонение числа баз, на которых искомый товар имеется в данный момент. Построить полигон полученного распределения.

    1. В урне 5 белых и 25 черных шаров. Вынули 3 шара. Случайная величина – число вы-

    1. нутых белых шаров. Составить закон распределения, функцию распределения, математическое ожидание, дисперсию и среднее квадратическое отклонение случайной величины. Построить по-лигон полученного распределения.

    1. Построить ряд распределения и функцию распределения числа попаданий м я ч о м в корзину при трех бросках, если вероятность попадания равна 0,4. Вычислить математическое ожидание, дисперсию и среднее квадратическое отклонение случайной величины. Построить по-

    1. лигон полученного распределения.

    2. 1 4 И з партии в 25 изделий, среди которых имеются 6

    3. бракованных, выбраны случайным образом 3 изделия для проверки и х качества. Определить закон, функцию распределения, математическое ожидание, дисперсию и среднее квадратическое отклонение количества бракованных среди выбранных. Построить полигон полученного распределения.

    1. В урне 5 белых и 25 черных шаров. Вынули 3 шара. Случайная величина – число вы-

    1. нутых черных шаров. Составить закон распределения, функцию распределения, математическое ожидание, дисперсию и среднее квадратическое отклонение случайной величины. Построить по-лигон полученного распределения.

    1. Построить ряд распределения и функцию распределения числа промахов при трех бросках мячом в корзину, если вероятность попадания равна 0,4. Вычислить математическое ожи-

    1. дание, дисперсию и среднее квадратическое отклонение случайной величины. Построить полигон полученного распределения.

    2. 1 7 И з партии в 25 изделий, среди которых имеются 6

    3. бракованных, выбраны случайным образом 3 изделия для проверки и х качества. Определить закон, функцию распределения, математическое ожидание, дисперсию и

    4. среднее квадратическое отклонение количества качественных среди выбранных. Построить полигон полученного распределения.

    5. Построить полигон полученного распределения.

    6. 19 С вероятностью попадания при одном выстреле 0,7 охотник стреляет по дичи до первого попадания, но успевает сделать не более 4 выстрелов. Дискретная случайная величина – число промахов. Определить закон, функцию распределения, математическое ожидание, дисперсию и среднее квадратическое отклонение случайной величины. Построить полигон полученного распределения.

    1. 2 стрелка делают по одному выстрелу в одну мишень, Beроятность попадания для первого стрелка при одном выстреле 0,5, для второго – 0,4. Дискретная случайная величина — число попаданий в мишень. Определить закон, функцию распределения, математическое ожидание, дисперсию и среднее квадратическое отклонение случайной величины. Построить полигон полученного распределения.

    1. В коробке имеются 7 карандашей, из которых 4 красные. Из этой коробки наудачу извлекаются 3 карандаша. Определить закон, функцию распределения, математическое ожидание, дисперсию и среднее квадратическое отклонение случайной величины, равной числу красных карандашей. Построить полигон полученного распределения.

    1. В коробке имеются 7 карандашей, из которых 4 красные. Из этой коробки наудачу извлекаются 3 карандаша. Определить закон, функцию распределения, математическое ожидание, дисперсию и среднее квадратическое отклонение случайной величины, равной числу некрасных карандашей. Построить полигон полученного распределения.

    1. Имеются 5 ключей, из которых только один подходит к замку. Найдите закон, функцию распределения, математическое ожидание, дисперсию и среднее квадратическое отклонение случайной величины, равной числу проб при открывании замка, если испробованный ключ в последующих опробованиях не участвует. Построить полигон полученного распределения.

    1. В партии из 10 деталей имеется 8 стандартных. Наудачу отобраны 2 детали. Составить закон распределения числа стандартных деталей среди отобранных. Найти функцию распределения, математическое ожидание, дисперсию и среднее квадратическое отклонение. Построить полигон полученного распределения.

    1. В коробке имеются 10 карандашей, из которых 4 синие. Из этой коробки наудачу извлекаются 3 карандаша. Определить закон, функцию распределения, математическое ожидание, дисперсию и среднее квадратическое отклонение случайной величины, равной числу синих карандашей. Построить полигон полученного распределения.

    1. Дискретная случайная величина – число девочек в семьях с 4 детьми. Предполагая равновероятными рождения мальчика и девочки найти закон, функцию распределения, математическое ожидание, дисперсию и среднее квадратическое отклонение количества девочек. Построить полигон полученного распределения.

    1. 2 стрелка делают по одному выстрелу в одну мишень, Beроятность попадания для первого стрелка при одном выстреле 0,6, для второго – 0,7. Дискретная случайная величина — число попаданий в мишень. Определить закон, функцию распределения, математическое ожидание, дисперсию и среднее квадратическое отклонение случайной величины. Построить полигон полученного распределения.

    1. 28 С вероятностью попадания при одном выстреле 0,8 охот-ник стреляет по дичи до первого попадания, но успевает сделать не более 4 выстрелов. Дискретная случайная величина – число промахов. Определить закон, функцию распределения, математическое ожидание, дисперсию и среднее квадратическое отклонение случайной величины. Построить полигон полученного распределения.

    2. Содержание отчета: отчет по практической работе должен содержать: рассуждения по решению задач, необходимые вычисления, ответ; вывод по работе

    3. Контрольные вопросы: 1 Что такое случайная величина? 2 Дискретная случайная вели-чина 3 Непрерывная случайная величина 4 Что такое закон распределения СВ? 5 Способы за-дания случайной величины 6 Что такое ряд распределения СВ? 7 Что называется многоуголь-ником распределения вероятности? 8 Функция распределения случайной величины 9 Что такое математическое ожидание? 10 Что называется дисперсией случайной величины? 11 Понятие среднего квадратического отклонения СВ.

    4. Практическая работа №23.

    5. Решение практических задач с применением вероятностных методов.

    6. Цель:

    7. Научиться решать практические задачи с применение вероятностных методов.

    8. Учебные задачи:

    1. Проверить навык вычисления вероятности.

    2. Научить решать задачи с практическим содержанием.

    3. Выработать навык применения формулы Бернулли и формулы Байеса.

    4. Учить рассуждать и логически мыслить,

    1. Краткие теоретические и учебно-методические материалы по теме практического занятия

    2. Пусть событие hello_html_3784c646.gif может наступить при условии появления одного из несовместных событий hello_html_3cdea70c.gif, hello_html_m1e789c2d.gif, …, hello_html_m4d6b48f6.gif, которые образуют полную группу. Пусть известны вероятности этих событий и условные вероятности hello_html_m5bb5ca35.gif, hello_html_m47e124da.gif, …, hello_html_6e95ad0d.gif события hello_html_3784c646.gif. В поставленных условиях вероятность события hello_html_3784c646.gif можно найти по формуле:

    3. hello_html_41c6a307.gif

    4. формулу называют формулой полной вероятности;

    5. события hello_html_3cdea70c.gif, hello_html_m1e789c2d.gif, …, hello_html_m4d6b48f6.gif называют гипотезами.

    6. Формула Байеса( Бейеса)

    7. Пусть H1,H2,…,Hn - полная группа событий и А   – некоторое событие. Тогда по формуле для условной вероятности

    8. hello_html_m253ec900.gif (1)

    9. Здесь P(Hk /A) – условная вероятность события (гипотезы) Hk или вероятность того, что Hk реализуется при условии, что событие А произошло.

    10. По теореме умножения вероятностей числитель формулы (1) можно представить в виде

    11. Phello_html_m288eb975.gif = Phello_html_66c5fa8f.gif= P(A /Hk) P(Hk)

    12. Для представления знаменателя формулы (1) можно использовать формулу полной вероятности P(A)hello_html_m68d682fd.gif

    13. Теперь из (1) можно получить формулу, называемую формулой Байеса: hello_html_m723ec17c.gif

    14. По формуле Байеса исчисляется вероятность реализации гипотезы Hk при условии, что событие А произошло. Формулу Байеса еще называют формулой вероятности гипотез. Вероятность P(Hk) называют априорной вероятностью гипотезы Hk, а вероятность P(Hk /A) – апостериорной вероятностью.

    15. Теорема. Вероятность гипотезы после испытания равна произведению вероятности гипотезы до испытания на соответствующую ей условную вероятность события, которое произошло при испытании, деленному на полную вероятность этого события.

    16. Формула Бернулли

    17. Схема Бернулли — это когда производится n однотипных независимых опытов, в каждом из которых может появиться интересующее нас событие A, причем известна вероятность этого события P(A) = p. Требуется определить вероятность того, что при проведении n испытаний событие A появится ровно k раз.

    18. Теорема Бернулли: Вероятность наступления ровно k успехов в n независимых повторениях одного и того же испытания находится по формуле:

    19. где p – вероятность «успеха», q = 1- p - вероятность «неудачи» в отдельном опыте.

    20. Вопросы для закрепления теоретического материала к практическому занятию:

    1. Что называют полной группой события?

    2. Дайте определение независимого события.

    3. Дайте определение условной вероятности.

    4. Дайте определение совместных событий.

    5. Дайте определение несовместных событий.

    6. Сформулируйте правило умножения вероятностей.

    7. Сформулируйте правило умножения вероятностей.

    8. Что называется факториалом числа n?

    9. Сформулируйте теорему умножения событий.

    10. Сформулируйте теорему сложения событий.

    11. Формула условной вероятности.

    12. Формула полной вероятности.

    1. Задания для практического занятия:

    2. 1.Из урны, в которой находится N белых и M черных шаров, вынимают наудачу R шара. Какова вероятность того, что R шаров окажутся черными.

      Вариант
      1. N

      1. М

      1. R

      1. 1)

      1. 12

      1. 7

      1. 2

      1. 2)

      1. 11

      1. 6

      1. 2

      1. 3)

      1. 10

      1. 5

      1. 3

      1. 4)

      1. 9

      1. 4

      1. 1

      1. 5)

      1. 8

      1. 3

      1. 2

      1. 6)

      1. 7

      1. 5

      1. 2

      1. 7)

      1. 6

      1. 4

      1. 2

      1. 8)

      1. 9

      1. 7

      1. 3

      1. 9)

      1. 10

      1. 5

      1. 2

      1. 10)

      1. 12

      1. 6

      1. 3

    1. В ящике находится М деталей (разложены в случайном порядке). Из них стандартных деталей N. Рабочий берет наудачу три детали. Найти вероятность того, что, по крайней мере, одна из взятых деталей окажется стандартной (событие А).

    Вариант
    1. 1)

    1. 2)

    1. 3)

    1. 4)

    1. 5)

    1. 6)

    1. 7)

    1. 8)

    1. 9)

    1. 10)

    1. М

    1. 20

    1. 21

    1. 22

    1. 13

    1. 16

    1. 18

    1. 20

    1. 12

    1. 11

    1. 9

    1. N

    1. 4

    1. 3

    1. 6

    1. 8

    1. 7

    1. 4

    1. 6

    1. 4

    1. 5

    1. 4

    1. Из 10 лотерейных билетов два выигрыш. Определить вероятность того, что среди взятых наудачу пяти билетов один выигрышный.

    2. В НИИ работает 120 человек, из них 70 знают английский язык, 80 – немецкий, 50 – оба языка. Найти вероятность того, что выбранный наудачу сотрудник не знает ни одного иностранного языка.

    3. Решить задачу по формуле Байеса. Из 10 учеников, которые пришли на экзамен по математике, трое подготовились отлично, четверо – хорошо, двое – удовлетворительно, один – не подготовился. В билетах 20 вопросов. Отлично подготовившиеся ученики могут ответить на все 20 вопросов, хорошо подготовившиеся – на 16, удовлетворительно – на 10 и не подготовившиеся – на 5 вопросов. Каждый ученик получает наугад 3 вопроса из 20. Приглашенный первым ученик ответил на все 3 вопроса. Найти вероятность того, что он отличник.

    4. Решить задачу по формуле Бернулли. Вероятность приема радиосигнала при каждой передаче равна 0,8. Найти вероятность того, что при пятикратной передаче сигнал будет принят ровно 4 раза.

    1. Баскетболист забрасывает мяч в корзину с вероятностью попадания Р =0,4. Что вероятнее: ожидать попадание трех мячей при четырех бросках или попадание четырех мячей при шести бросках?

    2. На остановке останавливаются 5 маршрутов автобусов, 3 маршрута троллейбусов и 2 маршрута трамваев. В нужном направлении следует 1 автобус, 2 трамвая и 2 троллейбуса. Найти вероятность того, что первым появится нужный транспорт.

    1. Найти вероятность того, что наудачу взятое двузначное число окажется кратным либо 3, либо 5, либо тому и другому одновременно.

    1. Инструкция по выполнению практического занятия

    2. Познакомьтесь с конспектами лекций и краткой теоретической справкой. При решении практических заданий, рассмотрите примеры.

    3. Пример 1. На контроль поступают детали с двух станков. Производительность станков не одинакова. На первом станке изготовляют hello_html_m34acbce4.gif всех деталей, на втором – hello_html_277cb65a.gif. Вероятность брака на первом станке hello_html_291eefaa.gif на втором – hello_html_me696c2c.gif. Найти вероятность того, что поступившая на контроль деталь бракованная.

    4. Решение. Событие hello_html_3784c646.gif – поступившая на контроль деталь бракованная.

    5. hello_html_3cdea70c.gif и hello_html_m1e789c2d.gif– события означающие, что деталь сделана соответственно на первом и втором станке.

    6. Тогда по условию задачи:

    7. hello_html_m293881c6.gif hello_html_4ccce219.gif

    8. hello_html_m379fde9e.gif hello_html_41bd780a.gif.

    9. Искомая вероятность:

    10. hello_html_m47f385f6.gif.

    11. Пример 2. В условиях задачи 1, проверенная деталь оказалась бракованной. Определить вероятность того, что она была изготовлена на первом станке.

    12. Решение. Искомая вероятность hello_html_m6c124744.gif–вероятность, что деталь изготовлена на первом станке, при условии что уже известно, что деталь бракованная.

    13. По формуле Бейеса:

    14. hello_html_m1b45e476.gif .

    15. Из примера 1: hello_html_m293881c6.gif; hello_html_4ccce219.gif; hello_html_m413c9d64.gif.

    16. Искомая вероятность:

    17. hello_html_514404d9.gif.

    18. Пример 3. Монета бросается 10 раз. Какова вероятность двукратного появления герба?

    19. Решение. Число опытов n=10, m= 2. Событие А – «успех» – выпадение герба. Тогда hello_html_60ae7a4.gif.

    20. hello_html_7284765d.gif

    21. Ответ: 0,04395.

    22. Порядок выполнения отчёта по практической работе:

    23. 1. Решите задачи.

    24. 2. Ответить на вопросы для закрепления теоретического материала (устно).

    25. 3. Оформить отчёт по практической работе.

    26. Образец отчёта по практической работе:

    27. Тема.

    28. Учебная цель.

    29. Название практической работы.

    30. Решение заданий практической работы.

    31. Ответы на вопросы для закрепления теоретического материала.

    32. Практическая работа № 24.

    33. Изображение пространственных фигур.

    34. Цель:
      О:

    • Повторить свойства параллельных прямых и плоскостей.

    • Свойства параллельного проецирования.

    • Научиться правильно изображать плоские фигуры и объёмные тела на плоскости.

    1. В:

    • Формирование грамотной математической речи, умения слушать, анализировать, строить логические цепочки, делать выводы, работать с чертежами.

    • Формирование трудовых навыков, умения распределять своё рабочее время на занятии, быстро,  грамотно и аккуратно оформлять записи в своих конспектах.

    • Формирование математического мировоззрения, математической культуры,  культуры речи, использование  математических терминов и символики.

    1. Р:

    • Формирование умения чётко и ясно излагать свои мысли, обсуждать и  корректировать  высказывания своих одногруппников.

    • Формирование интереса к предмету математики путём использования  формы занятия беседа-лекция-практикум, использования наглядности (моделей).

    1. Определить верность следующих высказываний:

    1. Верно ли, что через любую точку пространства можно провести множество прямых параллельных данной прямой?

    1. ОтветНеверно.

    2. По теореме о существовании прямой, параллельной данной прямой через точку пространства можно провести единственную прямую.

    1. Верно ли, что если одна из двух параллельных прямых пересекает плоскость, то и другая тоже пересекает эту плоскость?

    1. Ответ: Верно.

    2. По лемме о пересечении плоскости двумя параллельными прямыми, если одна из параллельных прямых пересекает данную плоскость, то и другая прямая пересекает эту плоскость.

    1. Верно ли, что две непересекающиеся прямые в пространстве параллельны?

    1. Ответ: неверно.

    2. В пространстве не имеют общих точек параллельные и скрещивающиеся прямые.

    1. Верно ли, что если две прямые параллельны некоторой плоскости, то они параллельны друг другу?

    1. Ответ: неверно.

    2. Эти прямые могут быть не только параллельными, но и пересекаться, а также они могут быть скрещивающимися.

    3. Мы приступили к изучению стереометрии – геометрии в пространстве. Как всегда нам необходимо уметь изображать геометрические фигуры, причем все чертежи мы по-прежнему выполняем на плоскости (на странице тетради, на доске и т.д.). Каким образом пространственную фигуру (например, куб) можно «уложить» в плоскость?

    4. Для решения этой задачи применяется метод параллельного проектирования. Выясним его суть на примере простейшей геометрической фигуры – точки.

    5. Итак, у нас есть геометрическая фигура в пространстве – точка А. (слайд 2)

    6. hello_html_52b69f84.png

    7. Выберем в пространстве произвольную плоскость (её мы будем называть плоскостью проекций) и любую прямую a пересекает (она задает направление параллельного проектирования). (слайд 3)

    8. hello_html_m58b69309.png

    9. Проведем через точку А прямую, параллельную прямой а. Точка А’ пересечения этой прямой с плоскостью и есть проекция точки А на плоскость . Точку А ещё называют прообразом, а точку А’ – образом. Если А, то А’ совпадает с А. (слайд 4)

    10. hello_html_420c4237.png

    11. Рассматривая любую геометрическую фигуру как множество точек, можно построить в заданной плоскости проекцию данной фигуры. Таким образом можно получить изображение (или «проекцию») любой плоской или пространственной фигуры на плоскости (см.рис.).

    12. Наглядным примером параллельного проектирования является отбрасываемая любым объектом(прообраз) в пространстве тень(образ) от солнечных лучей(направление параллельного проектирования) на Земле(плоскость проекций). (слайд 5)

    13. hello_html_3c5a32fb.png

    14. Примечание 1. При параллельном проектировании не выбирают направление параллельного проектирования параллельно плоскости проекции (самостоятельно обоснуйте почему). (слайд 6)

    15. hello_html_13af27ce.png

    16. Примечание 2. При параллельном проектировании плоских фигур не выбирают направление параллельного проектирования параллельно плоскости, которой принадлежит эта плоская фигура, т.к. получающаяся при этом проекция не отражает свойства данной плоской фигуры. (слайд 7)

    17. hello_html_e3cb38d.png

    18. Примечание 3. Если направление параллельного проектирования перпендикулярно плоскости проекций, то такое параллельное проектирование называется ортогональным (прямоугольным) проектированием. (слайд 8)

    19. hello_html_33f5b210.png

    20. Примечание 4. Если плоскость проекций и плоскость, в которой лежит данная фигура параллельны (||(АВС)), то получающееся при этом изображение… правильно – равно прообразу! (слайд 9)

    21. hello_html_m17a2c264.png

    22. Параллельное проектирование обладает свойствами:

    1. параллельность прямых (отрезков, лучей) сохраняется; (слайд 10)

    1. hello_html_48cc41ff.png

    2. hello_html_m7d521c41.png

    1. отношение длин отрезков, лежащих на параллельных или на одной прямой сохраняется; (слайд 11)

    1. hello_html_71aa8dec.png

    2. Если, например, АВ=2CD, то А’В’=2CD’ или

    3. 3) Линейные размеры плоских фигур(длины отрезков, величины углов) не сохраняются (исключение – см. примечание 4). (слайд 12)

    4. hello_html_2c6349d7.png

    5. Закрепление материала:

    6. Построим изображение куба: (слайд 13)

    7. hello_html_6dbf6e21.png

    8. Далее разберем примеры изображения некоторых плоских фигур: (слайд 14)

    9. hello_html_m276f84d0.png

    10. Слайд 15:

    11. hello_html_md2e8d93.png

    12. Слайд 16:

    13. hello_html_148942e3.png

    14. Слайд 17:

    15. hello_html_m637a1272.png

    16. Разберемся, как построить изображение правильного шестиугольника.

    17. Разобьем правильный шестиугольник на три части: прямоугольник FBCE и два равнобедренных треугольника ΔFAB и ΔCDE. Построим вначале изображение прямоугольника FBCE – произвольный параллелограмм FBCE. Осталось найти местоположение двух оставшихся вершин – точек A и D.

    18. Вспомнив свойства правильного шестиугольника, заметим, что: 1) эти вершины лежат на прямой, проходящей через центр прямоугольника и параллельной сторонам BC и FE; 2) OK=KD и ON=NA.

    19. Значит:

    1. Находим на изображении точку О и проводим через неё прямую, параллельную BC и FE, получив при этом точки N и K.

    2. Откладываем от точек N и K от центра О на прямой такие же отрезки – в итоге получаем две оставшиеся вершины правильного шестиугольника A и D. (слайд 18)

    1. hello_html_m1c4b539a.png

    2. 1. Что является параллельной проекцией отрезка, треугольника, прямоугольника, квадрата, окружности?

    3. 2. Какие величины не изменяются при параллельном проецировании? (длина отрезка, градусная мера углов, отношения длин отрезков, отношение площадей двух фигур)?

    4. 3. Может ли при параллельном проецировании параллелограмма получиться трапеция и наоборот?

    5. Попробуйте самостоятельно построить изображение правильного пятиугольника.

    6. Подсказка: разбейте фигуру на две части – равнобокую трапецию и равнобедренный треугольник, а затем воспользуйтесь некоторыми свойствами этих фигур и, конечно же, свойствами параллельного проектирования.

    7. hello_html_60a550b1.png

    8. Практическая работа № 25

    9. Представление о правильных многогранниках. Сечение куба, призмы, пирамиды.

    10. ПОРЯДОК ВЫПОЛНЕНИЯ РАБОТЫ:

    1. Ответить на контрольные вопросы:

    1. а) Что такое многогранник? Что такое грань многогранника, ребро, вершина?

    2. б) Что такое призма (основания призмы, боковые грани, ребра)?

    3. в) Перечислите свойства оснований, боковых ребер, боковых граней произвольной призмы.

    4. г) Что такое высота призмы? Что такое диагональ призмы?

    5. д) Какая призма называется прямой? Перечислите ее свойства.

    6. е) Какая призма называется правильной? Перечислите ее свойства.

    1. Изучить материал справочной таблицы «Правильные призмы».

    1. Выполнить задания теста и обучающей самостоятельной работы.

    1. Изучить условие заданий для практической работы.

    1. Оформить отчет о работе (при выполнении работы пользуйтесь материалами справочной таблицы).

    1. Правильные призмы

    2. Правильная треугольная призма

    3. Основание – правильный (равносторонний) треугольник

    4. Прямоугольник MCC1M1 – медианное сечение

    5. AA1 H - высота призмы, AB a - сторона основания,

    6. hello_html_m6669cde5.jpg

    7. CM - высота основания A 600
      1. ACM BCM 300

      1. a 2

      1. 3

      1. 3aH

      1. S осн

      1. CM

      1. a 3

      1. ;

      1. Sбок

      1. ;

      1. ;

      1. 4

      1. 2

      1. S полн 3aH

      1. a 2

      1. V S

      1. осн

      1. H

      1. 3

      1. ;

      1. 2

    8. hello_html_5f6cc97e.jpghello_html_5f7ab336.jpghello_html_m51c964b1.jpg

      1. AB a


      Правильная четырёхугольная призма
    9. Основание – правильный четырёхугольник (квадрат)

    10. Прямоугольник ACC1A1 – диагональное сечение

    11. hello_html_m33b9099d.jpg

    12. AA1 H - высота призмы, - сторона основания,

    13. AC1 – диагональ призмы, AB1 – диагональ боковой грани AC – диагональ основания

      • - угол между диагональю призмы и основанием

    • - угол между диагональю призмы и боковой гранью

        • - угол между диагональю боковой грани и основанием

    1. hello_html_m19f9961.jpg

    1. a 2
      1. H

      1. a

      1. AC1 2a 2 H 2

      1. cos

      1. sin

      1. sin

      1. a

      1. S

      1. a 2

      1. AB a 2

      1. H 2

      1. осн

      1. 1

      1. cos

      1. ;

    2. hello_html_69d769a6.jpghello_html_6cf49d98.jpg

    • AC a 2 H tg

    1. hello_html_m6d8991b5.jpg

    2. S
      1. бок

      1. 4aH

      1. S

      1. полн

      1. 4aH 2a 2

      1. .

    1. Правильная шестиугольная призма Основание – правильный шестиугольник

    2. Прямоугольники BEE1B1 и BDD1B1 – диагональные сечения; AA1 H - высота призмы, AB a - сторона основания; BE1 – большая диагональ призмы;

    3. hello_html_m1ce794c7.jpg

    4. BD1 – меньшая диагональ боковой грани;

    5. BE – большая диагональ основания, BD– меньшая диагональ основания;

      • - угол между большей диагональю призмы и основанием;

    • - угол между меньшей диагональю призмы и основанием;

    1. BE 2a ; BD a
      1. 2a

      1. H

      1. 3

      1. ;

      1. BE1 4a 2 H 2

      1. cos

      1. sin ;

      1. S осн

      1. 3a 2

      1. 3

      1. a 3

      1. H

      1. S

      1. 6aH ;

      1. BD 3a 2 H 2

      1. бок

      1. 1

      1. cos

      1. sin

      1. 2

      1. ;

      1. ;

      1. Sполн 6aH 3a 2

      1. 3

    2. hello_html_m3a35739f.jpghello_html_m117d2237.jpghello_html_5dd90ffd.jpghello_html_m7d653856.jpghello_html_m51c964b1.jpghello_html_m6bad023a.jpg

    1. ТЕСТ

    2. «Площадь поверхности призмы»

    1. Сторона основания правильной шестиугольной призмы равна 6 см, а

    1. большая диагональ призмы образует с основанием угол, равный 30 0 . Площадь полной поверхности призмы равна:

    2. hello_html_m7ad6c3a.jpg

      1. см 2 ;

      1. б) 288 см 2 ;

      1. в) 272

      1. 2 см 2 ; г) 272 см 2 .

      1. 3

      1. 2.

      1. ABCA B C

      1. - правильная

      1. треугольная призма, сторона основания

      1. 1

      1. 1

      1. 1

      1. которой равна 4 см, боковое ребро равно 3 см, точки E и F – середины

      1. ребер A1 C1

      1. и

      1. B1 C1 . Площадь сечения призмы плоскостью, проходящей

      1. через точки С, E и F, равно:

      1. б) 3 см 2 ; в) 4 см 2 ;

      1. а) 5 см 2 ;

      1. г) 2 3 см 2 .

    3. hello_html_m402b9f16.jpghello_html_m65c92061.jpg

    4. ВАРИАНТЫ ПРАКТИЧЕСКОЙ РАБОТЫ.

    5. Вариант 1.

    6. Площадь основания правильной четырехугольной призмы равна 625 см2. Высота призмы равна 14hello_html_87ffba2.jpghello_html_m29cab361.jpg2 см. Вычислите площадь ее диагонального сечения.

    7. Вариант 2.

    8. Диагональ основания правильной четырехугольной призмы равна 16 см, а диагональ ее боковой грани – 14 см. Вычислите длину: а) высоты призмы; б) диагонали призмы.

    9. Вариант 3.

    10. Сумма длин всех ребер правильной треугольной призмы равна 90 см. Длины ребер, имеющих общую точку, пропорциональны числам 3, 4, 3. Вычислите:

    11. а) площадь боковой грани призмы; б) длину диагонали боковой грани призмы.

    12. Вариант 4.

    13. Диагональ боковой грани правильной треугольной призмы равна 6 дм и наклонена к плоскости основания под углом 30 0 . Вычислите площадь: а) основания призмы; б) боковой грани призмы.

    14. Вариант 5.

    15. Основание прямой призмы – прямоугольный треугольник, катеты которого равны 7 см и 24 см. Угол между диагональю большей боковой грани и плоскостью основания призмы равен 45 0 . Вычислите: а) длину высоты призмы; б) площадь меньшей боковой грани призмы.

    16. Вариант 6.

    17. Основание прямой призмы ABCA1 B1 C1 - равнобедренный треугольник (АВ = АС = 7 см). Высота призмы равна 6 см. Диагональ боковой грани ВСC1 B1 равна 15 см. Вычислите: а) площадь основания призмы; б) длину диагонали грани ABB1 A1 .

    18. Вариант 7.

    19. Основание прямой призмы – ромб со стороной 8 см и острым углом 60 0 .

    20. Высота призмы равна 12 см. Вычислите: а) длины диагоналей призмы; б) площади диагональных сечений.

    21. Вариант 8.

    22. Основание прямой призмы – ромб. Диагонали призмы равны 8 см и 5 см. высота ее – 2 см. Вычислите: а) длину стороны основания; б) площадь основания призмы.

    23. Вариант 9.

    24. Основание наклонной призмы ABCA1 B1 C1 - правильный треугольник, сторона которого равна 24 см. Вершина A1 проектируется в центр треугольнике АВС. Угол между боковым ребром призмы и плоскостью ее основания равен 45 0 . Вычислите: а) длину бокового ребра призмы; б) расстояние между основаниями призмы.

    25. Вариант 10.

    26. Диагональ правильной четырехугольной призмы равна а. Угол между этой диагональю и плоскостью основания вдвое больше, чем угол между нею и боковым ребром. Вычислите площадь боковой поверхности призмы.

    27. Вариант 11.

    28. Высота правильной треугольной призмы равна 16 см. Диагональ боковой грани наклонена к плоскости основания под углом 45 0 . Вычислите площадь полной поверхности призмы.

    29. Вариант 12.

    30. Высота правильной треугольной призмы ABCA1 B1 C1 равна 20 см. Угол между плоскостями основания и сечения, проведенного через ребро ВС и вершину A1 , равен 45 0 . Вычислите площадь полной поверхности призмы.

    31. Вариант 13.

    32. Площадь боковой поверхности правильной треугольной призмы равна сумме площадей ее оснований. Вычислите длину бокового ребра призмы, если сторона ее основания равна 6 см.

    33. Вариант 14.

    34. Основание прямой призмы – прямоугольная трапеция, основания и высота которой равны соответственно 9 см, 14 см и 12 см. Боковое ребро призмы равно 20 см. Вычислите площадь:

    35. а) большей боковой грани; б) полной поверхности призмы.

    36. Вариант 15.

    37. Основание прямой призмы – треугольник, стороны которого равны 10 см, 10 см и 12 см. Высота призмы равна высоте этого треугольника. Вычислите площадь: а) основания призмы; б) боковой поверхности призмы.

    38. Практическая работа № 26.

    39. Решение задач на нахождение элементов тел вращения.

    40. Цель: 1) Систематизация знаний и умений по теме.

    41. 2) Развитие практических навыков нахождения элементов тел вращения.

    42. 3) Развитие практических навыков применения формул для вычисления площадей

    43. поверхностей тел вращения.

    44. 4) Развитие навыков решения задач с практическим содержанием.

    45. Указания:

    1. Каждую задачу сопровождайте чертежом.

    2. Чертежи выполняйте карандашом и по линейке.

    3. В ходе решения записывайте формулы, которыми пользуйтесь.

    4. Решение оформляйте аккуратно и грамотно.

    1. Задания:

      1.Высота цилиндра равна 6 см, площадь его осевого сечения 60 см2. Вычислите длину окружности основания цилиндра.
      1. 1.Диагональ осевого сечения цилиндра равна 12 см, а угол между этой диагональю и плоскостью основания цилиндра 30°. Вычислите площадь основания цилиндра.

      1. 2.Угол между образующей конуса и его высотой 60°. Вычислите площадь боковой поверхности конуса, если его высота равна 6 см.

      1. 2.Угол между образующей конуса и плоскостью его основания равен 30°, длина окружности основания - 12 см. Вычислите длину образующей конуса.

      1. 3.Шар пересекает плоскость на расстоянии 9 см от центра, площадь сечения

      2. 1600 см2. Определите радиус шара.

      1. 3.Чему равна площадь сферы, если площадь большого круга равна 24 см2?

      1. 4.Малярный валик имеет длину, равную 230 мм, диаметр основания 50 мм. Какова площадь поверхности, которую окрасит маляр за один полный прокат валика? Сколько таких полных прокатов совершит маляр при окраске за смену 200 м2 поверхности?

      1. 4.Котёл имеет форму цилиндра длиной 4,2 м и диаметром основания 1,2 м. Сколько квадратных метров железа пойдёт на его изготовление, если на швы добавляется 12%?

      1. 5.Найдите образующую усечённого конуса, если радиусы оснований равны 9 см и 5 см, а высота равна 4 см.

      1. 5.Во что обойдётся окраска конического шпиля здания, диаметр основания которого 9,8 м? Угол между образующими в осевом сечении 60°, окраска 1 м2 стоит 1,15 тыс. рублей.

      1. 6.Полуцилиндрический свод подвала имеет длину 8 м и диаметр 6 м. Определите поверхность свода подвала.

      1. 6.Шар радиуса 20 см пересечён плоскостью, находящейся на расстоянии 10 см от центра. Найдите площадь сечения.

      1. 7.Сколько потребуется кожи для изготовления покрышки футбольного мяча диаметром 20 см, если на обрезки и швы расходуется 8% сверх расчётной площади?

      1. 7.Требуется окрасить бак цилиндрической формы с диаметром основания 4 м и высотой 5 м. Сколько понадобится краски, если на 1 м2 расходуется 200 г краски?

  10. Практическая работа № 27.

  11. Решение задач на нахождение объемов тел и площадей их поверхностей.

  12. Обучающая цель: научить применять формулы объемов и площадей поверхностей тел вращения в практических ситуациях.

  13. Развивающая цель: продолжить формирование умений применять математические знания в практических ситуациях, описанных условием задачи.

  14. Воспитательная цель: продолжить воспитание мотивации учения, раскрывая профессиональную и практическую значимость изучаемого материала.

  15. Задача 1. Требуется оштукатурить две колонны одинаковой высоты, но с различными поперечными сечениями: круглым и квадратным. Наружный диаметр круглого сечения и сторона наружного квадрата равны 30 см. На какую колонну расходуется штукатурки больше, и во сколько раз?

  16. Решение:

  17. раз

  18. Задача 2. Во что обойдется окраска конического шпиля здания, диаметр окружности основания которого 8,8 м? Угол между образующими в осевом сечении 600 , окраска 1 м2 по ЕНИР стоит 55 руб.

  19. s Дано: конус Решение:

  20. D=8,8 м Зп= Sбок. k = R l k

  21. 300 <ASС = 600 SOC

  22. 8,8 k = 55 руб./м2 l = 2R (катет против угла в 300 )

  23. Найти: Зп Зп = Rl k = R 2R k =2R2 k

  24. Зп 23,144,4255 6686,9 руб.

  25. 4.4

  26. A 0 C

  1. Назовите фигуру, соответствующую данным формулам нахождения объемов:

  1. R2 H 1/3 R2 H R H 4/3R3 H

  2. (цилиндр) (конус)

  1. Образующая конуса равна 5 см, высота 3 см. Найти радиус. Отв. 4 см.

  2. Найти площадь поверхности шара, диаметр которого равен 10 см.Отв100 π см2

  3. Найти объем цилиндра, если его осевое сечение квадрат и сторона квадрата

  1. 4 см. Отв. 16 π см3.

  2. А) Решение задач.

  3. Задача 1 (№ 2). Рабочий оштукатуривает вручную колонну улучшенной штукатуркой. Сколько времени ему понадобится, чтобы оштукатурить колонну высотой 6 м, диаметром 1 м, соблюдая норму времени 0,79 ч на 1 м2?

  4. Дано: цилиндр Решение:

  5. Н = 6 м t = Sбок. k =2RH k =

  6. D = 1 м = D H k 3.14160.79

  7. К = 0,79 ч/м2 14.9 (час.) 14 час. 54 мин.

  8. Найти: t

  9. Задача 2 (№ 3).Маляру требуется покрыть поверхность общей площадью 200 м2. Внутренний диаметр резервуара краскопульта ручного действия 178 мм, высота 715 мм. Известно, что при двукратном покрытии расходуется 480 г краски на 1 м2. Рассчитайте, сколько раз и каким количеством водной краски придется наполнять рабочему резервуар, чтобы не оставалось излишков. Указание: Принять массу 1 л водной краски равной 1 кг.

  10. Дано: цилиндр краскопульт Решение:

  11. S = 200 м2 Надо на 200м2 краски:

  12. D = 178 мм = 1,78 дм mоб. = S К 200 0,48 96 (кг.)

  13. H = 715 мм = 7,15 дм Объем краскопульта:

  14. К = 480 г/ м2 = 0,48 кг/ м2 Vпр. = R2 H 3,14 0,892 7,15 17,8 (дм3)

  15. ρ = 1 кг/дм3 Масса краски к краскопульте

  16. Найти: n, mоб. Mкр. = ρ Vкр. 17,8 (кг).

  17. 96 : 17,8 5,39. В пять объёмов 96 кг не умещается.

  18. Значит надо 6 раз наполнять резервуар по 16кг ( 96:6 = 16 кг)

  19. Задача 3(№5).

  20. На строительных площадках песок хранят в штабелях. После приёмки влажный песок уложили в штабель конической формы, размеры которого оказались следующими: длина

  21. окружности основания 32 м, длина по откосу 7 м. Определить объём принимаемого песка, учитывая скидку на влажность воздуха 15.

  22. S Дано: конус Решение:

  23. C = 32м V = 0,85Vк =

  24. 7 l = 7м = 0,851/3R2 H

  25. n = 15 % C = 32

  26. Найти: V 2R = 32

  27. 16/ R = 32/2 = 16/ 5,09м

  28. ASO

  29. A O B

  30. V = 0.851/33.145.0924,8 110.64 (м3 )

  31. б) Самостоятельная работа.

  32. I вариант

  33. Задача № 6. Радиус круга, лежащего в основании конуса, равен 3 дм , а угол между образующей и основанием составляет 600.

  34. Найдите:

  35. S а) образующую конуса

  36. б) высоту конуса

  37. в) площадь боковой поверхности конуса

  38. г) площадь полной поверхности конуса

  39. д) угол между образующими осевого сечения конуса

  40. 600 е) объем конуса

  41. А о В

  42. Дано: R = 3дм Решение:

  43. SAO = 600 а) l = 2R = 6 (дм)

  44. Найти: б) ASO

  45. а) l г) Sполн. H2 = AS2 - AO2 = 62 - 32 = 36 – 9 = 27

  46. б) H д) ASB H = 27 = 93 = 33 (дм)

  47. в) Sбок е) Vк в) Sбок. = R l = 36 = 18(дм2)

  48. г) Sполн. = Sбок + Sосн. = 18 + 9 = 27 (дм2)

  49. д) 600

  50. е) V = 1/3 Sосн H = 1/39 3√3 = 9√3 (дм3)

  51. II вариант.

  52. Задача №7. Длина окружности основания цилиндра составляет 8см, высота равна 6см.

  53. Найдите:

  54. а) радиус основания цилиндра

  55. б) площадь основания цилиндра

  56. в) площадь боковой поверхности цилиндра

  57. г) площадь полной поверхности цилиндра

  58. д) площадь осевого сечения цилиндра

  59. е) объем цилиндра

  60. Решение:

  61. а) C = 2R = 8

  62. Дано: цилиндр R = 8/2 = 4/

  63. D C C = 8 б) Sосн. = R2 = 16/2 = 16/ (см2)

  64. H = 6см в) Sбок. = 2RH= 24/6= 48(см2)

  65. Найти: г) Sполн.= Sбок + Sосн. = 48+ 16/(см2)

  66. 6 а) R г) Sполн. д) Sос. сеч. = DH= 8/6= 48/ (см2)

  67. б) Sосн. д) Sос. сеч. е) V= R2H= 16/26 = 96/ (см3)

  68. в) Sбок е) V

  69. A O B

  70. в) Практическая индивидуальная работа по карточкам– заданиям.

  71. Задача. Провести необходимые измерения штангенциркулем и вычислить площадь полной поверхности и объем данных тел вращения (трое).

  72. Информация о конусе ( устные сообщения учащихся с показом слайдов).

  73. 1) По статистике на Земле ежегодно гибнет от разрядов молний 6 человек на 1000 000 жителей (чаще в южных странах). Этого бы не случилось, если бы везде были громоотводы, т.к. вокруг громоотвода образуется конус безопасности. Чем выше громоотвод, тем больше объем такого конуса. Люди пытаются спрятаться от разрядов под

  74. деревом, но дерево не проводник, на нем заряды накапливаются, поэтому дерево становится источником напряжения.

  75. 2) В биологии есть понятие «конус нарастания». Это верхушка побега и корня растений, состоящая из клеток образовательной ткани.

  76. 3) «Конусами» называется и семейство морских моллюсков подкласса переднежаберных. Раковина коническая (2-16 см), ярко окрашенная. Конусов свыше 500 видов. Живут в тропиках и субтропиках, являются хищниками, имеют ядовитую железу. Укус конусов очень болезнен. Известны смертельные случаи. Раковины используются как сувениры, украшения.

  1. Задача. К электромешалке для приготовления гипсоопилочной мастики прилагаются четыре цилиндрических бачка. За каждый цикл работы заполняются все четыре бачка, каждый из бачков имеет диаметр 350 мм и высоту 430 мм. Продолжительность цикла приготовления мастики, включая засыпку и выгрузку, составляет примерно 10 мин. Какое количество мастики можно приготовить в течение одного часа непрерывной работы?

  1. ДАНО: 4 цилиндра Vобщ = πRH·4·6D2/4H46

  2. D=350мм =3,5дм D2 H26 3,143,524,36

  3. H=430мм =4,3дм 3,1412,254,36 992,4 (дм3)

  4. t=10 мин 992,4 (л) 0,99 м3

  5. tоб =1ч = 60мин

  6. Найти: Vобщ.

  7. 1) № 50. Сколько олифы потребуется для окраски внешней поверхности 100 ведер, имеющих форму усеченного конуса с диаметром оснований 25см и 30см и образующей 27,5см, если на 1м2 требуется 150 гр. олифы?

  8. ДАНО: усеченный конус РЕШЕНИЕ:

  9. n=100 m=(Sбок + Sосн) · К · 100

  10. d1=25см Sбок + Sосн=П (R + r ) · e +Пr2 3,14·

  11. D=30см ·((15+12,5)·27,5+12,52) 3,14(27,52+

  12. е=27,5см +12,52) 3,14(756,25+156,25) 3,14 ·

  13. к=150г/м2 · 912,5 2865см2 0,2865м2

  14. НАЙТИ: m m 0,2865·150·100 4297(г) 4,3кг.

  15. Практическая работа №28.

  16. Векторы.

  17. Цель работы: Закрепить знания и совершенствовать умения поданной теме.

  18. Ход работы:

  19. 1. Ответить на контрольные вопросы:

  20. 1). Определение вектора

  21. 2). Определение коллинеарных векторов

  22. 3). Определение сонаправленных векторов

  23. 4) Определение противоположно направленных векторов

  24. 5) Определение равных векторов

  25. 6) Определение компланарных векторов

  26. 2. Выполнить контрольное задание.

  27. Образец выполнения заданий.

  28. 1. В прямоугольном параллелепипеде отмечены вектора . Найти среди них коллинеарные, сонаправленные, противоположно направленные и равные.

  29. Решение:hello_html_4c8ced24.jpg

  30. 1. Коллинеарные:

  31. 2. Сонаправленные:

  32. 3. Противоположно направленные:

  33. 4. Равные:

  34. 2. Упростить выражение

  35. Решение:

  1. Диагонали куба пересекаются в точке О. Найти число k, если

  1. Решение:hello_html_100663fd.jpg

  2. а) Т.к.

  3. О б) Т.к.

  4. 4. В параллелепипеде ABCD точка М середина АВ. Разложите вектор

  5. Решение:hello_html_m6b3da963.jpg

  6. М

  7. 5. В кубе ABCD точка k - середина . Разложите вектор

  8. Решение:hello_html_1892f58e.png

  9. Представим вектор в виде суммы следующих векторов:

  10. Тогда получим

  11. Выполните самостоятельно:

    1. Дан прямоугольный параллелепипед ABCDA1B1C1D1 и вектора. Найдите среди них:
  12. 1) Коллинеарные

  13. 2) Сонаправленные

  14. 3) Противоположно направленные

  15. 4)Равные

    1. 1. Дан прямоугольный параллелепипед ABCDA1B1C1D1 и вектора. Найдите среди них:

    2. 1) Коллинеарные

    3. 2) Сонаправленные

    4. 3) Противоположно направленные

    5. 4)Равные

    1. 2. Упростить выражение:

    1. 2. Упростить выражение:

    1. 3. Диагонали куба ABCDA1B1C1D1 пересекаются в точке О. Найти число k, если:

    2. =k

    1. 3. Диагонали куба ABCDA1B1C1D1 пересекаются в точке О. Найти число k, если:

    2. =k

    1. 4. В параллелепипеде ABCDA1B1C1D1, точки М и N середины АВ и А1D1. Разложите вектор по

    1. 4. В параллелепипеде ABCDA1B1C1D1, точки М и N середины АВ и А1D1. Разложите вектор по

    1. 5. Точка К – середина ребра ВС тетраэдра ABCD. Разложите вектор по =, =,=.

    1. 5. Основанием пирамиды является параллелограмм ABCD. Точка О является вершиной пирамиды. Разложите вектор по =, =,=

  16. Практическая работа № 29-30.

  17. Использование координат и векторов при решении математических и прикладных задач.

  18. Цель: овладеть навыками использования правил действий над векторами в координатной формах.

  19. Теоретические сведения.

  20. Определение: «Вектор — это направленный отрезок». Длиной вектора называется длина этого отрезка. Обозначается:  или 

  21. Равными называются векторы, имеющие одинаковые длины и одинаковое направление. Это значит, что вектор можно перенести параллельно себе в любую точку плоскости.
    Единичным называется вектор, длина которого равна 1. Нулевым — вектор, длина которого равна нулю, то есть его начало совпадает с концом.

  22. Сложение векторов

  23. Для сложения векторов есть два способа.

  24. 1. Правило параллелограмма. Чтобы сложить векторы  и , помещаем начала обоих в одну точку. Достраиваем до параллелограмма и из той же точки проводим диагональ параллелограмма. Это и будет сумма векторов  и .

  25. Помните басню про лебедя, рака и щуку? Они очень старались, но так и не сдвинули воз с места. Ведь векторная сумма сил, приложенных ими к возу, была равна нулю.

  26. 2. Второй способ сложения векторов — правило треугольника. Возьмем те же векторы  и . К концу первого вектора пристроим начало второго. Теперь соединим начало первого и конец второго. Это и есть сумма векторов  и .

  27. По тому же правилу можно сложить и несколько векторов. Пристраиваем их один за другим, а затем соединяем начало первого с концом последнего.

  28. Представьте, что вы идете из пункта А в пункт В, из В в С, из С в D, затем в Е и в F. Конечный результат этих действий — перемещение из А в F.

  29. При сложении векторов  и  получаем:

  30. ,

  31. Вычитание векторов

  32. Вектор  направлен противоположно вектору . Длины векторов  и равны.

  33. Теперь понятно, что такое вычитание векторов. Разность векторов  и  — это сумма вектора  и вектора .

  34. Умножение вектора на число

  35. При умножении вектора  на число k получается вектор, длина которого в k раз отличается от длины . Он сонаправлен с вектором , если k больше нуля, и направлен противоположно , если k меньше нуля.

  36. Скалярное произведение векторов

  37. Векторы можно умножать не только на числа, но и друг на друга.

  38. Скалярным произведением векторов называется произведение длин векторов на косинус угла между ними.

  39. Если векторы перпендикулярны, их скалярное произведение равно нулю. 
    А вот так скалярное произведение выражается через координаты векторов  и 

  40. Из формулы для скалярного произведения можно найти угол между векторами:

  41. Свойства векторов.

  42. Определение. Два вектора  и  называются коллинеарными, если они расположены на параллельных прямых или на одной прямой.

  43. Нулевой вектор  коллинеарен любому вектору.

  44. Теорема 1. Два ненулевых вектора   и коллинеарны,  когда они пропорциональны т.е.  = kk – скаляр.

  45. Определение. Три вектора , ,  называются компланарными, если они параллельны некоторой плоскости или лежат в ней.

  46. Теорема 2. Три ненулевых вектора , ,  компланарны,  когда один из них является линейной комбинацией двух других, т.е.  = k + lk ,l – скаляры.

  47. Задания для самостоятельной работы

  48. 1. Даны координаты двух смежных вершин А и В квадрата АВСD: А(а; 0) и В(0; а). Найдите координаты остальных вершин квадрата.

  49. Сколько решений имеет задача?

  50. 2. Даны две смежные вершины параллелограмма АВСD: А(–1; 3) и В(2; 1). Найдите координаты двух других вершин параллелограмма, если известно, что диагональ АС параллельна оси Ох, а диагональ ВD параллельна оси Оу.

  51. 3. Треугольник задан координатами своих вершин: А(1; 0; –1), В(3; 2; 0) и С(–2; 4; 3). Является ли данный треугольник прямоугольным; равнобедренным?

  52. 4. Векторы и служат диагоналями параллелограмма АВСD. Выразите векторы и через векторы и .

  53. 5. Точки К и М служат серединами сторон АВ и CD выпуклого четырехугольника АВСD. Выразите вектор через векторы и .

  54. 6. Треугольник АВС задан координатами своих вершин: А(1; 1), В(5; 4) и С(13; –2). Определите координаты и длины векторов , и , где АМ – медиана, BD – биссектриса и СН – высота треугольника АВС.

  55. 7. Одна из вершин параллелепипеда АВСDА1В1С1D1 находится в точке А(1; 2; 3), а концы выходящих из нее ребер – в точках В(9; 6; 4), D(3; 0; 4) и А1(5; 2; 6). Найдите:

  56. а) длину диагонали АС1 этого параллелепипеда;

  57. б) угол, образуемый этой диагональю с ребром АВ;

  58. в) объем параллелепипеда;

  59. г) площадь основания АВСD и высоту параллелепипеда, опущенную на это основание.

  60. 8. Четырехугольник задан координатами своих вершин: А(0; 0), В(0; 4), С(2; 4) и D(7; 0). Можно ли в него вписать окружность?

  61. Примечание. Если суммы противоположных сторон четырехугольника равны, то в него можно вписать окружность.

  62. 9. В треугольнике АВС проведена медиана АМ. Найдите ее длину, если известны координаты вершин треугольника: А(5; –4), В(-1; 2) и С(5; 1).

  63. 10. Вычислите площадь квадрата, две смежные вершины которого А(3; –7) и В(–1; 4).

  64. 11. Найдите координаты точки С, делящей отрезок АВ в отношении 3 : 2, считая от вершины А, если А(–2; 1), В(8; 6).

  65. 12. Однажды Лебедь, Рак и Щука …

  66. Воз расположен в точке О пересечения медиан треугольника АВС, а Лебедь, Рак и Щука – в вершинах треугольника. Определите суммарное воздействие на воз, т.е. .

  67. 13. Лежат ли точки А(2; 4; 1), В(3; 7; 5) и С(4; 10; 9) на одной прямой?

  68. Указание. I способ. Используйте условие коллинеарности векторов.

  69. II способ. Сравните расстояния АВ, ВС и АС.

  70. 14. В треугольнике АВС известно: (3; –4) и (1; 5). Вычислите длину высоты СН этого треугольника.

  71. Указание. С помощью векторного произведения векторов определите площадь треугольника, а затем воспользуйтесь формулой площади треугольника, которая выражается через высоту.

  72. 15. Треугольник АВС задан координатами своих вершин: А(1; 1), В(5; 4) и С(13; –2) – условия задачи 6 (см. выше). Вычислите площадь треугольника АВС различными способами:

  73. а) с помощью основания и высоты (используйте результат задачи 6);

  74. б) с помощью формулы ;

  75. в) с помощью формулы Герона;

  76. г) с помощью векторного произведения.

  77. 16. Зная одну из вершин треугольника АВС А(2; –5; 3) и векторы, совпадающие с двумя его сторонами (4; 1; 2) и (3; –2; 5), найдите координаты остальных вершин треугольника и вектор .

  78. Указание. Используйте формулу определения координат вектора по координатам его начала и конца.

  79. 17. Будут ли компланарны векторы и :

  80. а) , (7; –18; 2);

  81. б) , (3; 1; –1)?

  82. Указание. Используйте условие компланарности векторов (равенство нулю смешанного произведения векторов).

  83. Информационное обеспечение обучения

  84. Перечень рекомендуемых учебных изданий, Интернет-ресурсов, дополнительной литературы

  85. Основные источники:

  1. Башмаков М.И. Математика: учебник для учреждений нач. и сред.проф. образования. – М.: Издательский центр «Академия», 2012– 256 с.

  2. Башмаков М.И. Математика: задачник для учреждений нач. и сред.проф. образования. – М.: Издательский центр «Академия», 2012– 416 с.

  3. Пехлецкий И.Д. Математика: учебник для студ. образоват. учреждений сред.проф. образования М.: Издательский центр «Академия», 2008 г.

  1. Дополнительные источники

  2. Алимов Ш.А. и др. Алгебра и начала анализа. 10 (11) кл. – М., 2010.

  3. Атанасян Л.С. и др. Геометрия. 10 (11) кл. – М., 2010.

  4. Башмаков М.И. Алгебра и начала математического анализа (базовый уровень). 10кл. – М., 2013.

  5. Башмаков М.И. Алгебра и начала математического анализа (базовый уровень). 11кл. – М., 2013.

  6. Башмаков М.И. Математика (базовый уровень). 10—11кл. – М., 2005.

  7. Башмаков М.И. Математика: 10 кл. Сборник задач: учеб. пособие. – М., 2013.

  8. Башмаков М.И. Математика: учебник для 10 кл. – М., 2013.

  9. Колмогоров А.Н. и др. Алгебра и начала анализа. 10 (11) кл. – М., 2000.

  10. Колягин Ю.М. и др. Математика (Книга 1). – М., 2010.

  11. Колягин Ю.М. и др. Математика (Книга 2). – М., 2010.

  12. Луканкин Г.Л., Луканкин А.Г. Математика. Ч. 1: учебное пособие для учреждений начального профессионального образования. – М., 2011.

  13. Пехлецкий И.Д. Математика: учебник. – М., 2011.

  14. Смирнова И.М. Геометрия. 10 (11) кл. – М., 2011.

  15. Интернет-ресурсы

  16. Математика в Открытом колледже http://www.mathematics.ru

  17. Math.ru: Математика и образование htpp:\\www.math.ru

  18. Московский центр непрерывного математического образования http:\\mccme.ru

  19. Allmath.ru—вся математика в одном месте http:\\www.allmath.ru

  20. EqWorld: Мир математических уравнений http:\\eqworld.ipmnet.ru

  21. Средняя математическая интернет-школа http:\\www.bymath.net

  22. Геометрический портал htpp:\\ www.neive.by.ru

  23. Графики функций http:\\graphfunk.narod.ru

Просмотрено: 0%
Просмотрено: 0%
Скачать материал
Скачать материал "МЕТОДИЧЕСКИЕ УКАЗАНИЯ по выполнению практических работ по дисциплине ОУД.03 «Математика: алгебра и начала математического анализа, геометрия» 1 курс специальность 35.02.01"

Методические разработки к Вашему уроку:

Получите новую специальность за 6 месяцев

Руководитель реабилитационного подразделения

Получите профессию

Копирайтер

за 6 месяцев

Пройти курс

Рабочие листы
к вашим урокам

Скачать

Краткое описание документа:

Целью данных методических рекомендаций является организация преподавателем эффективной работы студентов на практических занятиях по дисциплине ОУД.03 «Математика: алгебра и начала математического анализа, геометрия» как средства, способствующего повышению качества образовательного процесса. Практическая работа является одним из видов учебных занятий студентов, выполняемых под руководством преподавателя.

Основные цели практических работ:

- систематизация и закрепление знаний и практических умений студентов полученных при изучении на уроке;

- углубление и расширение теоретических знаний, формирование умений использовать справочную документацию, дополнительную литературу;

- развитие познавательных способностей и активности студентов, творческой инициативы, самостоятельности, ответственности и организованности;

- формирование самостоятельного мышления;

- развитие исследовательских умений.

Скачать материал

Найдите материал к любому уроку, указав свой предмет (категорию), класс, учебник и тему:

6 664 334 материала в базе

Скачать материал

Другие материалы

Вам будут интересны эти курсы:

Оставьте свой комментарий

Авторизуйтесь, чтобы задавать вопросы.

  • Скачать материал
    • 30.11.2017 3995
    • DOCX 3.6 мбайт
    • Оцените материал:
  • Настоящий материал опубликован пользователем Чудоквасова Галина Анатольевна. Инфоурок является информационным посредником и предоставляет пользователям возможность размещать на сайте методические материалы. Всю ответственность за опубликованные материалы, содержащиеся в них сведения, а также за соблюдение авторских прав несут пользователи, загрузившие материал на сайт

    Если Вы считаете, что материал нарушает авторские права либо по каким-то другим причинам должен быть удален с сайта, Вы можете оставить жалобу на материал.

    Удалить материал
  • Автор материала

    • На сайте: 9 лет и 5 месяцев
    • Подписчики: 9
    • Всего просмотров: 126556
    • Всего материалов: 35

Ваша скидка на курсы

40%
Скидка для нового слушателя. Войдите на сайт, чтобы применить скидку к любому курсу
Курсы со скидкой

Курс профессиональной переподготовки

Менеджер по туризму

Менеджер по туризму

500/1000 ч.

Подать заявку О курсе

Курс повышения квалификации

Внедрение системы компьютерной математики в процесс обучения математике в старших классах в рамках реализации ФГОС

36/72 ч.

от 1700 руб. от 850 руб.
Подать заявку О курсе
  • Сейчас обучается 139 человек из 52 регионов
  • Этот курс уже прошли 492 человека

Курс повышения квалификации

Методика обучения математике в основной и средней школе в условиях реализации ФГОС ОО

72 ч. — 180 ч.

от 2200 руб. от 1100 руб.
Подать заявку О курсе
  • Сейчас обучается 430 человек из 73 регионов
  • Этот курс уже прошли 5 552 человека

Курс повышения квалификации

Особенности подготовки к сдаче ЕГЭ по математике в условиях реализации ФГОС СОО

36 ч. — 180 ч.

от 1700 руб. от 850 руб.
Подать заявку О курсе
  • Сейчас обучается 188 человек из 55 регионов
  • Этот курс уже прошли 1 700 человек

Мини-курс

Инновационные технологии для бизнеса

4 ч.

780 руб. 390 руб.
Подать заявку О курсе

Мини-курс

Эволюция и современное состояние искусства

6 ч.

780 руб. 390 руб.
Подать заявку О курсе

Мини-курс

Психология расстройств пищевого поведения

3 ч.

780 руб. 390 руб.
Подать заявку О курсе
  • Сейчас обучается 161 человек из 53 регионов
  • Этот курс уже прошли 95 человек